24.11.2013 Views

Invariantes. Todo lo que debías saber para la OMA

Invariantes. Todo lo que debías saber para la OMA

Invariantes. Todo lo que debías saber para la OMA

SHOW MORE
SHOW LESS

You also want an ePaper? Increase the reach of your titles

YUMPU automatically turns print PDFs into web optimized ePapers that Google loves.

Matemática e <strong>Invariantes</strong><br />

1


Indice<br />

1. Pregunta y Advertencia 4<br />

2. El Engaño 5<br />

2.1. Introducción . . . . . . . . . . . . . . . . . . . . . . . . . . . . . . . . 6<br />

2.2. Primeros pasos . . . . . . . . . . . . . . . . . . . . . . . . . . . . . . 8<br />

2.3. Luces . . . . . . . . . . . . . . . . . . . . . . . . . . . . . . . . . . . . 14<br />

2.4. Piedras . . . . . . . . . . . . . . . . . . . . . . . . . . . . . . . . . . . 18<br />

2.5. Fichas . . . . . . . . . . . . . . . . . . . . . . . . . . . . . . . . . . . 20<br />

2.6. Un poco de Geometría Proyectiva . . . . . . . . . . . . . . . . . . . . 24<br />

2.7. Cartas . . . . . . . . . . . . . . . . . . . . . . . . . . . . . . . . . . . 31<br />

2.8. Rompecabezas . . . . . . . . . . . . . . . . . . . . . . . . . . . . . . . 36<br />

2.9. Pizarrones, Tab<strong>la</strong>s y Grupos . . . . . . . . . . . . . . . . . . . . . . . 39<br />

2.10. Senku . . . . . . . . . . . . . . . . . . . . . . . . . . . . . . . . . . . 43<br />

2.11. Ranas . . . . . . . . . . . . . . . . . . . . . . . . . . . . . . . . . . . 46<br />

2.12. Monovariantes . . . . . . . . . . . . . . . . . . . . . . . . . . . . . . . 48<br />

2.13. Un problema y 2 soluciones . . . . . . . . . . . . . . . . . . . . . . . 51<br />

2.14. Diamante Azteca . . . . . . . . . . . . . . . . . . . . . . . . . . . . . 53<br />

2.15. Pulgas . . . . . . . . . . . . . . . . . . . . . . . . . . . . . . . . . . . 59<br />

3. Apéndices 61<br />

3.1. Sumas infinitas . . . . . . . . . . . . . . . . . . . . . . . . . . . . . . 62<br />

3.2. Sumas de potencias . . . . . . . . . . . . . . . . . . . . . . . . . . . . 65<br />

3.3. Sucesiones recurrentes . . . . . . . . . . . . . . . . . . . . . . . . . . 67<br />

3.4. Permutaciones . . . . . . . . . . . . . . . . . . . . . . . . . . . . . . . 69<br />

3.5. Mene<strong>la</strong>o . . . . . . . . . . . . . . . . . . . . . . . . . . . . . . . . . . 71<br />

4. Problemas 75<br />

4.1. Algunos problemas <strong>para</strong> pensar . . . . . . . . . . . . . . . . . . . . . 76<br />

3


1. Pregunta y Advertencia<br />

Advertencia: Cada solución <strong>que</strong> escribía despertaba el mismo miedo, <strong>que</strong> iban<br />

a leer <strong>la</strong> solución antes de pensar el problema y así no só<strong>lo</strong> perderían <strong>la</strong> oportunidad de<br />

pensar el problema sino <strong>que</strong> entenderían menos <strong>la</strong> solución. Por eso esta advertencia,<br />

es mejor pensar un problema por algún tiempo y <strong>que</strong> no salga <strong>que</strong> leer su solución en<br />

5 minutos y olvidarse de él. Piensen cada problema, no se apresuren a leer su solución.<br />

Pregunta: “Hacer matemática no es cuestión de acumu<strong>la</strong>r ideas, trucos o teoremas<br />

<strong>para</strong> <strong>que</strong> cada vez <strong>que</strong> se nos presente un problema repasemos <strong>la</strong> lista hasta<br />

dar con el teorema indicado.....ni siquiera es importante conocer toda c<strong>la</strong>se de teoremas,<br />

trucos o técnicas....“ ¿Por qué entonces escribir algo <strong>que</strong> parece un manual<br />

<strong>para</strong> resolver problemas usando invariantes? ¿No parece esto tratar de listar todos <strong>lo</strong>s<br />

invariantes habidos y por haber?<br />

4


2. El Engaño<br />

5


2.1. Introducción<br />

Supongamos <strong>que</strong> tenemos un objeto <strong>que</strong> va cambiando de acuerdo a ciertas reg<strong>la</strong>s.<br />

Nos podemos preguntar:<br />

¿En qué puede transformarse?<br />

¿Cuánto tardará en llegar a cierto estado?<br />

¿Puede cambiar indefinidamente?<br />

¿Puede transformarse en algo en particu<strong>la</strong>r?<br />

Los invariantes nos ayudan a responder estas preguntas, <strong>la</strong> idea es mirar <strong>lo</strong> <strong>que</strong><br />

no cambia a medida <strong>que</strong> nuestro objeto va tranformándose. Veamos un ejemp<strong>lo</strong>, el<br />

problema es de <strong>la</strong> XI Olimpíada Nacional Argentina.<br />

Se tienen 3 hormigas en <strong>lo</strong>s vértices de un cuadrado. En cada turno, una<br />

hormiga se puede mover en dirección <strong>para</strong>le<strong>la</strong> a <strong>la</strong> recta <strong>que</strong> determinan<br />

<strong>la</strong>s otras 2. ¿Es posible <strong>que</strong> después de algunos turnos <strong>la</strong>s hormigas ocupen<br />

3 puntos medios de <strong>lo</strong>s <strong>la</strong>dos del cuadrado?<br />

Figura 1: La hormiga en C puede ir hasta D.<br />

6


Solución: No, no es posible. Notemos <strong>que</strong> el área del triángu<strong>lo</strong> <strong>que</strong> forman <strong>la</strong>s 3<br />

hormigas no varía con <strong>lo</strong>s turnos, pues cada vértice se mueve en forma <strong>para</strong>le<strong>la</strong> al <strong>la</strong>do<br />

opuesto 1 . Si el cuadrado inicial tenía área 1, entonces 3 vértices forman un triángu<strong>lo</strong><br />

de área 1 y 3 puntos medios forman un triángu<strong>lo</strong> con area 1 . Entonces no es posible<br />

2 4<br />

<strong>que</strong> <strong>la</strong>s hormigas terminen en 3 puntos medios de <strong>lo</strong>s <strong>la</strong>dos del cuadrado. ‡<br />

Figura 2: El área nunca cambia<br />

En este caso el objeto era <strong>la</strong> posición de <strong>la</strong>s 3 hormigas y el “Invariante“ el área<br />

del triángu<strong>lo</strong> <strong>que</strong> formaban.<br />

1 Lo <strong>que</strong> estamos usando es <strong>que</strong> si tenemos 2 triángu<strong>lo</strong>s ABC y ABD con AB//CD entonces<br />

área(ABC) = área(ABD)<br />

7


2.2. Primeros pasos<br />

Para empezar, algunos ejemp<strong>lo</strong>s sencil<strong>lo</strong>s. En todos <strong>lo</strong>s problemas tenemos algo<br />

<strong>que</strong> va cambiando de acuerdo a ciertas reg<strong>la</strong>s, <strong>lo</strong> <strong>que</strong> tenemos <strong>que</strong> hacer es mirar con<br />

cuidado y encontrar algo <strong>que</strong> no cambia.<br />

• En el pizarrón hay 10 signos + y 5 signos −. En cada turno se debe<br />

elegir dos de el<strong>lo</strong>s, borrar<strong>lo</strong>s y poner un + si eran iguales y un − si eran<br />

distintos. Probar <strong>que</strong> después de 14 turnos <strong>que</strong>dará un signo − so<strong>la</strong>mente.<br />

Si pensamos a cada signo + como un 1 y a cada signo − como un −1, al borrar 2<br />

números <strong>lo</strong>s debemos reemp<strong>la</strong>zar por su producto, entonces el producto de todos <strong>lo</strong>s<br />

números en el pizarrón no cambia. Como inicialmente el producto era 1 10 (−1) 5 = −1,<br />

entonces luego de 14 turnos cuando só<strong>lo</strong> <strong>que</strong>da un número, éste debe ser −1. ‡<br />

• En el pizarrón hay escritos 2 números, en cada turno se borran <strong>lo</strong>s 2<br />

números y si éstos eran a y b entonces se reemp<strong>la</strong>zan por 2a − b y 2b − a.<br />

Si inicialmente estaban escritos el 1458 y 1460. ¿Es posible <strong>que</strong> luego de<br />

varios turnos <strong>lo</strong>s números en el pizarrón sean el 715 y el 1024?<br />

Como 2a − b + 2b − a = a + b entonces <strong>la</strong> suma de <strong>lo</strong>s 2 números escritos en el<br />

pizarrón nunca cambia. Al principio ésta es 1458 + 1460 = 2918, como 715 y 1024 no<br />

suman 2918 entonces <strong>lo</strong>s números en el pizarrón nunca serán 715 y 1024. ‡<br />

• Se tiene un tablero de 8 × 8 de ajedrez pintado de b<strong>la</strong>nco y negro de <strong>la</strong><br />

forma usual. En cada turno se pueden intercambiar 2 columnas o 2 fi<strong>la</strong>s.<br />

¿Es posible <strong>lo</strong>grar <strong>que</strong> todas <strong>la</strong>s casil<strong>la</strong>s negras estén en <strong>la</strong> mitad izquierda<br />

del tablero y <strong>la</strong>s casil<strong>la</strong>s b<strong>la</strong>ncas en <strong>la</strong> mitad derecha?<br />

Notemos <strong>que</strong> <strong>la</strong> cantidad de casil<strong>la</strong>s negras en cada columna nunca varía, como<br />

inicialmente había 4 casil<strong>la</strong>s negras en cada columna entonces no se pueden poner<br />

todas <strong>la</strong>s negras en <strong>la</strong> mitad izquierda ya <strong>que</strong> tendríamos 8 negras en cada una de <strong>la</strong>s<br />

primeras 4 columnas. ‡<br />

8


• Inicialmente se tiene <strong>la</strong> terna (3, 4, 12). En cada paso se puede tomar <strong>lo</strong>s<br />

números a y b y cambiar<strong>lo</strong>s por 0,6a − 0,8b y 0,8a + 0,6b. ¿Se puede llegar<br />

a (4, 6, 12)?<br />

Notemos <strong>que</strong>:<br />

(0,6a−0,8b) 2 +(0,8a+0,6b) 2 = 0,36a 2 −0,48ab+0,64b 2 +0,64a 2 +0,48ab+0,36b 2 = a 2 +b 2<br />

Entonces <strong>la</strong> suma de <strong>lo</strong>s cuadrados de <strong>lo</strong>s 3 números nunca cambia. Es decir <strong>que</strong> si<br />

llegamos a <strong>la</strong> terna (a, b, c), luego a 2 +b 2 +c 2 = 3 2 +4 2 +12 2 = 169, pero 4 2 +6 2 +12 2 =<br />

196 > 169. Entonces no se puede llegar a (4, 6, 12). ‡<br />

A veces no es tan sencil<strong>lo</strong> encontrar “<strong>la</strong> cuenta correcta“....<br />

• Un círcu<strong>lo</strong> está dividido en 6 sectores (como en <strong>la</strong> figura), <strong>lo</strong>s números<br />

1, 0, 1, 0, 0, 0 están escritos en el<strong>lo</strong>s. En cada turno se puede sumarle 1 a<br />

dos sectores adyacentes. ¿Es posible <strong>lo</strong>grar <strong>que</strong> todos <strong>lo</strong>s sectores tengan<br />

el mismo número?<br />

Si a 1 , a 2 , a 3 , a 4 , a 5 , a 6 son <strong>lo</strong>s números escritos entonces S = a 1 −a 2 +a 3 −a 4 +a 5 −a 6<br />

permanece invariante. Inicialmente S = 2, si todos <strong>lo</strong>s números se vuelven iguales<br />

entonces sería S = 0. ¡Lo <strong>que</strong> es absurdo! De donde nunca <strong>lo</strong>s 6 sectores tendrán el<br />

mismo número. ‡<br />

9


• Si se tiene el par {x, y} se <strong>lo</strong> puede cambiar por el par {y, x} , {x − y, y}<br />

o {x + y, y}. ¿Se puede obtener el par {819, 357} a partir del par {1, 2}?<br />

¿Qué pares se puede obtener a partir del {1, 2}?<br />

Notemos <strong>que</strong> el máximo común divisor de <strong>lo</strong>s 2 números <strong>que</strong> forman el par no<br />

varía, pues (x, y) = (y, x) = (x − y, y) = (x + y, y). Como (1, 2) = 1 y (819, 357) = 21<br />

entonces no es posible llegar de {1, 2} a {819, 357}.<br />

Si desde {1, 2} se puede llegar a {a, b} entonces (a, b) = 1. Para ver <strong>que</strong> desde<br />

{1, 2} se puede llegar a cualquier par de números coprimos notemos <strong>que</strong> si de un<br />

par se puede llegar a otro, entonces desde el último se puede volver al primero (esto<br />

se debe a <strong>que</strong> <strong>la</strong>s operaciones son reversibles). Entonces tenemos <strong>que</strong> ver <strong>que</strong> desde<br />

{a, b} se puede llegar a {1, 2}. Procedemos de <strong>la</strong> siguiente forma, dado {a, b} siempre<br />

hagamos <strong>que</strong> el primer número sea más grande <strong>que</strong> el segundo y luego le restamos el<br />

segundo al primero. Repetimos <strong>lo</strong> anterior hasta <strong>que</strong> obtengamos un 0. Para ver <strong>que</strong><br />

efectivamente esto debe ocurrir supongamos <strong>lo</strong> contrario y notemos <strong>que</strong> <strong>lo</strong>s números<br />

siempre se mantienen no negativos pero en cada turno se van haciendo más y más<br />

chicos <strong>lo</strong> <strong>que</strong> es absurdo. Si llegamos a {0, m} entonces m = (0, m) = (a, b) = 1 de<br />

donde m = 1. Es fácil ver <strong>que</strong> desde {0, 1} se puede llegar a {1, 2}. ‡<br />

Es muy común <strong>que</strong> si bien <strong>la</strong> suma o el producto no permanecen invariantes, sí <strong>lo</strong><br />

hace su paridad o su resto en <strong>la</strong> división por 3 (o por algún otro número elegido de<br />

forma adecuada).<br />

• En una is<strong>la</strong> hay 15 camaleones azules, 17 rojos y 19 verdes. Cuando 2<br />

camaleones de distinto co<strong>lo</strong>r se encuentran ambos cambian al tercer co<strong>lo</strong>r.<br />

¿Es posible <strong>que</strong> en algún momento todos <strong>lo</strong>s camaleones se vuelvan del<br />

mismo co<strong>lo</strong>r?<br />

Pongámosle a cada camaleón azul un sombrero con un 1, a cada camaleón rojo<br />

uno con un 2 y a cada camaleón verde uno con un 3. Cuando un camaleón cambia de<br />

co<strong>lo</strong>r, también cambia su número (al correspondiente según el nuevo co<strong>lo</strong>r). Cada vez<br />

<strong>que</strong> se encuentran 2 camaleones y cambian de co<strong>lo</strong>r, <strong>la</strong> suma de <strong>lo</strong>s números aumenta<br />

o disminuye en 3 o permanece igual. Entonces si bien no permanece invariante, sí <strong>lo</strong><br />

hace su resto en <strong>la</strong> división por 3. Inicialmente <strong>la</strong> suma es 15 + 2 × 17 + 3 × 19 =<br />

15 + 34 + 57 = 106 <strong>que</strong> tiene resto 1 en <strong>la</strong> división por 3. Si todos <strong>lo</strong>s camaleones se<br />

volvieran del mismo co<strong>lo</strong>r, digamos al correspondiente al número k, entonces <strong>la</strong> suma<br />

sería 51k <strong>que</strong> es múltip<strong>lo</strong> de 3 (hay 51 camaleones en total). ¡Absurdo! Entonces <strong>lo</strong>s<br />

camaleones no pueden volverse todos del mismo co<strong>lo</strong>r. ‡<br />

10


• En un pizarrón hay cierta cantidad de letras a, b, c. En cada turno se<br />

puede reemp<strong>la</strong>zar una a y una b por una b, una a y una c por una c,<br />

una b y una c por una a, dos a por una a, dos b por una c o dos c por<br />

una b. El objetivo es dejar una so<strong>la</strong> letra. Probar <strong>que</strong> sin importar en qué<br />

orden realicemos <strong>la</strong>s operaciones, <strong>la</strong> letra <strong>que</strong> <strong>que</strong>de al final siempre será<br />

<strong>la</strong> misma.<br />

Reemp<strong>la</strong>cemos a cada a por un 0, a cada b por un 1 y a cada c por un 2. Notemos<br />

<strong>que</strong> <strong>la</strong>s reg<strong>la</strong>s <strong>para</strong> borrar letras y escribir otra se pueden reemp<strong>la</strong>zar por <strong>la</strong> siguiente<br />

más simple, se eligen 2 números y se <strong>lo</strong>s reemp<strong>la</strong>za por el resto de su suma en <strong>la</strong><br />

división por 3. Es fácil ver <strong>que</strong> <strong>la</strong> suma de todos <strong>lo</strong>s números escritos tendrá siempre<br />

el mismo resto en <strong>la</strong> división por 3 y éste será el número <strong>que</strong> <strong>que</strong>de al final. Entonces<br />

sin importar el orden en <strong>que</strong> se realizaron <strong>la</strong>s operaciones, <strong>la</strong> última letra (o número)<br />

será siempre <strong>la</strong> misma. ‡<br />

• Nos es dado un tablero de 2 × 2 con números en sus casil<strong>la</strong>s. En cada<br />

paso podemos realizar una de <strong>la</strong>s siguientes operaciones:<br />

-Sumar a una fi<strong>la</strong> un múltip<strong>lo</strong> de <strong>la</strong> otra.<br />

-Sumar a una columna un múltip<strong>lo</strong> de <strong>la</strong> otra.<br />

Si inicialmente había dos 1 en una diagonal y 0 en <strong>la</strong>s otras 2 casil<strong>la</strong>s. ¿Es<br />

posible conseguir una fi<strong>la</strong> de 0?<br />

a<br />

c<br />

b<br />

d<br />

1 0<br />

0 1<br />

→<br />

∗ ∗<br />

0 0<br />

Para cada tablero con <strong>lo</strong>s números a, b, c, d en sus casil<strong>la</strong>s como muestra <strong>la</strong> figura,<br />

vamos a l<strong>la</strong>mar a ad − bc su “determinante“. Es fácil ver <strong>que</strong> éste no varía con <strong>la</strong>s<br />

operaciones permitidas, pues si por ejemp<strong>lo</strong> sumamos a <strong>la</strong> segunda fi<strong>la</strong> k veces <strong>la</strong><br />

primera entonces<br />

a(d + kb) − b(c + ka) = ad + akb − bc − bka = ad − bc<br />

Inicialmente el determinante es 1 y si conseguimos una fi<strong>la</strong> de ceros sería 0. ¡Absurdo!<br />

Entonces no es posible conseguir una fi<strong>la</strong> de ceros. ‡<br />

11


Algunos problemas <strong>para</strong> pensar<br />

1. Sea n un entero positivo impar y supongamos <strong>que</strong> en el pizarrón están escritos<br />

<strong>lo</strong>s números 1, 2, .., 2n. En cada turno se deben elegir 2 de el<strong>lo</strong>s, digamos a y b,<br />

se <strong>lo</strong>s borra y se escribe |a − b|. Probar <strong>que</strong> luego de 2n − 1 pasos, el número<br />

escrito será impar.<br />

2. Un dragón tiene 100 cabezas. Un caballero le puede cortar 15, 17, 20 o 5 cabezas<br />

pero al hacer<strong>lo</strong>, al dragón le crecen 24, 2, 14 y 17 cabezas nuevas respectivamente.<br />

¿Puede el caballero dejar al dragón sin cabezas?<br />

3. Tenemos 50 peones en un tablero de 10 × 10 de forma <strong>que</strong> 25 de el<strong>lo</strong>s están<br />

en <strong>la</strong> esquina inferior izquierda de 5 × 5 del tablero y <strong>lo</strong>s 25 restantes están en<br />

<strong>la</strong> esquina superior derecha de 5 × 5 (uno por casil<strong>la</strong>). En cada turno podemos<br />

elegir un peón y hacer<strong>lo</strong> saltar por sobre cualquier otro peón vecino a <strong>la</strong> casil<strong>la</strong><br />

siguiente si ésta se encuentra vacía (cada peón a <strong>lo</strong> sumo tiene 8 vecinos). ¿Es<br />

posible <strong>que</strong> luego de varios turnos <strong>lo</strong>s 50 peones estén en <strong>la</strong> mitad izquierda del<br />

tablero?<br />

4. Borramos el primer dígito de 7 1996 y se <strong>lo</strong> sumamos al número obtenido. Se repite<br />

esta operación hasta obtener un número de 10 dígitos. Probar <strong>que</strong> el número de<br />

10 dígitos obtenido tendrá 2 dígitos iguales.<br />

5. En el pizarrón están escritos <strong>lo</strong>s números 1, 1, 1, .., 1 . En cada turno se deben<br />

2 3 n<br />

elegir 2 de el<strong>lo</strong>s, digamos a y b, borrar<strong>lo</strong>s y escribir en su lugar ab + a + b. ¿Qué<br />

número <strong>que</strong>da después de n − 1 turnos?<br />

6. Sean a 1 , a 2 , .., a n tales <strong>que</strong> a i = ±1 y S = a 1 a 2 a 3 a 4 +a 2 a 3 a 4 a 5 +...+a n a 1 a 2 a 3 = 0.<br />

Probar <strong>que</strong> n es múltip<strong>lo</strong> de 4.<br />

7. En cada vértice de un hexágono hay una semil<strong>la</strong>, en cada turno se pueden<br />

escoger 2 de el<strong>la</strong>s y mover<strong>la</strong>s 1 vértice en direcciones opuestas. Probar <strong>que</strong> no<br />

es posible llevar todas <strong>la</strong>s semil<strong>la</strong>s a un so<strong>lo</strong> vértice.<br />

8. Nos es dado un tablero de 3 × 3 con números en sus casil<strong>la</strong>s. En cada paso<br />

podemos realizar una de <strong>la</strong>s siguientes operaciones:<br />

-Sumar a una fi<strong>la</strong> un múltip<strong>lo</strong> de otra.<br />

-Sumar a una columna un múltip<strong>lo</strong> de otra.<br />

a b c<br />

d e f<br />

g h i<br />

12


Probar <strong>que</strong> <strong>la</strong> siguiente cantidad permanece invariante:<br />

a(ei − fh) − b(di − fg) + c(dh − eg)<br />

Nota: Este número es el determinante de <strong>la</strong> matriz (tablero) anterior. Es fácil<br />

ver <strong>que</strong> si intercambiamos 2 columnas entre sí entonces el determinante sigue<br />

siendo el mismo pero con distinto signo. ¿Cuál será el determinante de una<br />

matriz de n × n?<br />

9. Tenemos un tablero de 3 × 3 con números enteros y le podemos aplicar <strong>la</strong>s<br />

mismas operaciones <strong>que</strong> en el problema anterior. Un menor de 2 × 2 del tablero<br />

es el determinante del tablero <strong>que</strong> se obtiene al borrar una fi<strong>la</strong> y una columna.<br />

Probar <strong>que</strong> el máximo común divisor de todos <strong>lo</strong>s menores de 2 × 2 no varía.<br />

13


2.3. Luces<br />

A veces, si bien el primer invariante <strong>que</strong> se nos ocurre no funciona, <strong>lo</strong> único <strong>que</strong><br />

tenemos <strong>que</strong> hacer es usar <strong>la</strong> misma cuenta pero en un conjunto más chico. Veamos<br />

algunos ejemp<strong>lo</strong>s.<br />

En cada punto de <strong>la</strong> figura hay una luz. En cada turno podemos elegir una<br />

fi<strong>la</strong> o columna y cambiar el estado de todas <strong>la</strong>s luces en el<strong>la</strong>. Inicialmente<br />

todas <strong>la</strong>s luces están apagadas salvo <strong>la</strong> marcada en <strong>la</strong> figura. ¿Es posible<br />

<strong>lo</strong>grar apagar todas <strong>la</strong>s luces?<br />

No, no es posible. La solución es sencil<strong>la</strong>, alcanza notar <strong>que</strong> siempre habrá una<br />

cantidad impar de luces prendidas.<br />

Dicho de otra forma, pongamos un 1 por cada luz apagada y un −1 por cada<br />

luz prendida. Cada vez <strong>que</strong> cambiamos el estado de una luz estamos multiplicando<br />

su número por −1. Como en cada turno cambiamos el estado de 4 luces entonces<br />

multiplicamos 4 números por −1, de donde el producto de <strong>lo</strong>s 16 números permanece<br />

invariante.<br />

Si <strong>lo</strong>gramos apagar todas <strong>la</strong>s luces el producto sería 1, <strong>lo</strong> <strong>que</strong> es imposible ya <strong>que</strong><br />

inicialmente era −1. ‡<br />

¿Qué pasa si en el problema anterior también nos permitimos cambiar el<br />

estado de cualquier diagonal de cualquier <strong>lo</strong>ngitud? (En particu<strong>la</strong>r podemos<br />

cambiar el estado de cualquier esquina). ¿Podemos apagar todas <strong>la</strong>s<br />

luces?<br />

14


El argumento anterior no funciona ya <strong>que</strong> el producto no permanece invariante,<br />

pero..... ¿Por qué multiplicar todos <strong>lo</strong>s números? ¿Por qué no buscar un conjunto más<br />

pe<strong>que</strong>ño cuyo producto sí permanece invariante?<br />

Si consideramos <strong>la</strong>s 8 luces marcadas de negro en <strong>la</strong> figura, cada fi<strong>la</strong>, columna<br />

o diagonal cambia el estado de 0 o 2 de estas 8 luces, luego el producto de estos 8<br />

números debe ser “Invariante“. Como inicialmente es -1 nunca podremos apagar todas<br />

<strong>la</strong>s luces ya <strong>que</strong> esto supone llegar a un producto = 1. ‡<br />

En cada punto de <strong>la</strong> figura hay una luz, en cada turno se puede elegir<br />

un <strong>la</strong>do o diagonal del pentágono y cambiar el estado de todas <strong>la</strong>s luces<br />

<strong>que</strong> están en esa línea. Inicialmente todas <strong>la</strong>s luces están apagadas. ¿Es<br />

posible prender <strong>la</strong>s 10 luces?<br />

15


Como antes, reemp<strong>la</strong>cemos <strong>la</strong>s luces apagadas por un 1 y <strong>la</strong>s prendidas por -1. Esta<br />

vez estamos en <strong>la</strong> situación opuesta a <strong>la</strong> anterior. El producto permanece invariante<br />

pero el inicial es el mismo al <strong>que</strong> <strong>que</strong>remos llegar, <strong>lo</strong> <strong>que</strong> no nos lleva a ningún absurdo.<br />

La estrategia es <strong>la</strong> misma, buscar un conjunto más pe<strong>que</strong>ño tal <strong>que</strong> el producto<br />

permanezca invariante pero <strong>que</strong> sea distinto el inicial y el final.<br />

Tomemos <strong>lo</strong>s 5 puntos exteriores de <strong>la</strong> figura, <strong>lo</strong>s vértices del pentágono. Como<br />

cada <strong>la</strong>do o diagonal toca exactamente 2 de estos 5 puntos entonces en cada turno el<br />

producto de estos 5 números es multiplicado por (−1) 2 = 1, es decir <strong>que</strong> permanece<br />

invariante.<br />

Inicialmente el producto es 1, pero si conseguimos tener <strong>la</strong>s 10 luces prendidas<br />

sería −1. ¡Absurdo! Entonces no es posible <strong>lo</strong>grar <strong>que</strong> todas estén prendidas. ‡<br />

En <strong>lo</strong>s ejemp<strong>lo</strong>s anteriores vimos 2 situaciones de cómo buscar el invariante adecuado<br />

cuando el más natural no nos sirve, ya sea por<strong>que</strong> no permanezca invariante o<br />

por<strong>que</strong> no nos lleve a ningún absurdo. Veamos un ejemp<strong>lo</strong> más.<br />

Se tienen 8 luces en fi<strong>la</strong>. En cada turno se pueden elegir 3 luces consecutivas<br />

y cambiar su estado. Si inicialmente todas <strong>la</strong>s luces están apagadas salvo<br />

<strong>la</strong>s 2 en <strong>lo</strong>s bordes, ¿es posible apagar todas <strong>la</strong>s luces?<br />

Pongamos un 1 por cada luz apagada y un -1 por cada luz prendida. La operación<br />

permitida es multiplicar por −1 cualesquiera 3 números seguidos. Notemos <strong>que</strong> si<br />

consideramos el producto total, éste varía y da <strong>lo</strong> mismo en <strong>la</strong> situación inicial y <strong>la</strong><br />

final. Es por el<strong>lo</strong> <strong>que</strong> debemos buscar un conjunto más pe<strong>que</strong>ño de luces.<br />

Si consideramos <strong>la</strong>s luces marcadas en <strong>la</strong> figura. Entonces en cada operación vamos<br />

a cambiar el estado de 2 luces, <strong>que</strong> es par. Como inicialmente el producto es −1 no<br />

podemos apagar todas ya <strong>que</strong> esto nos lleva a un producto = 1. ‡<br />

16


Algunos problemas <strong>para</strong> pensar<br />

1. Se tienen 16 luces ordenadas en 4 fi<strong>la</strong>s y 4 columnas, en cada turno se pueden<br />

elegir 3 luces seguidas (en <strong>la</strong> misma fi<strong>la</strong> o en <strong>la</strong> misma columna) y cambiar el<br />

estado de <strong>la</strong>s 3. Si inicialmente están todas prendidas... ¿Es posible apagar <strong>la</strong>s<br />

16 luces?<br />

2. Se tienen 100 luces ordenadas en 10 fi<strong>la</strong>s y 10 columnas, en cada turno se pueden<br />

elegir 4 luces seguidas (en <strong>la</strong> misma fi<strong>la</strong> o en <strong>la</strong> misma columna) y cambiar su<br />

estado. Si inicialmente están todas prendidas... ¿Es posible apagar <strong>la</strong>s 100 luces?<br />

3. (Torneo de <strong>la</strong>s Ciudades 2003) En cada casil<strong>la</strong> de un tablero de 4 × 4 hay un<br />

signo + o un signo −. La operación permitida es elegir una casil<strong>la</strong> y cambiar el<br />

signo de esa casil<strong>la</strong> junto con todos <strong>lo</strong>s de sus vecinas (<strong>que</strong> tienen un <strong>la</strong>do en<br />

común). Determinar cuántos tableros diferentes se pueden obtener.<br />

4. (Torneo de <strong>la</strong>s Ciudades 2004)<br />

a) En cada casil<strong>la</strong> de un tablero de m × n con m, n ≥ 2 hay un signo + o un<br />

signo −, <strong>la</strong> operación permitida es cambiar todos <strong>lo</strong>s signos de una fi<strong>la</strong> o todos<br />

<strong>lo</strong>s signos de una columna. Diremos <strong>que</strong> un tablero es irreducible si es imposible<br />

mediante operaciones permitidas transformar<strong>lo</strong> en uno con signos + en todas<br />

<strong>la</strong>s casil<strong>la</strong>s. Demostrar <strong>que</strong> todo tablero irreducible contiene un cuadrado de<br />

2 × 2 <strong>que</strong> es un tablero irreducible de 2 × 2.<br />

b) En cada casil<strong>la</strong> de un tablero de m × n con m, n ≥ 4 hay un signo + o<br />

un signo −, <strong>la</strong> operación permitida es cambiar todos <strong>lo</strong>s signos de una fi<strong>la</strong> o<br />

todos <strong>lo</strong>s signos de una columna o todos <strong>lo</strong>s signos de una diagonal. Diremos<br />

<strong>que</strong> un tablero es irreducible si es imposible mediante operaciones permitidas<br />

transformar<strong>lo</strong> en uno con signos + en todas <strong>la</strong>s casil<strong>la</strong>s. Demostrar <strong>que</strong> todo<br />

tablero irreducible contiene un cuadrado de 4 × 4 <strong>que</strong> es un tablero irreducible<br />

de 4 × 4.<br />

5. Supongamos <strong>que</strong> tenemos n lám<strong>para</strong>s y algunos interruptores. Cada interruptor<br />

cambia el estado de algunas lám<strong>para</strong>s. Inicialmente hay algunas lám<strong>para</strong>s prendidas.<br />

El objetivo es apretar algunos de <strong>lo</strong>s interruptores <strong>para</strong> apagar<strong>la</strong>s todas.<br />

Probar <strong>que</strong> si esto no es posible entonces se pueden elegir algunas lám<strong>para</strong>s de<br />

forma <strong>que</strong> no importa qué interruptores apretemos, siempre habrá una cantidad<br />

impar de lám<strong>para</strong>s prendidas.<br />

17


2.4. Piedras<br />

El problema <strong>que</strong> sigue fue propuesto por Konstechiv <strong>para</strong> el II Torneo de <strong>la</strong>s<br />

Ciudades en 1981. Nosotros presentamos una pe<strong>que</strong>ña modificación del problema<br />

original.<br />

Consideremos todos <strong>lo</strong>s puntos del p<strong>la</strong>no con coordenadas enteras no negativas.<br />

Inicialmente tenemos una piedra en (0, 0), una en (0, 1) y otra en<br />

(1, 0). En cada paso debemos elegir una piedra en un punto (m, n) y si <strong>lo</strong>s<br />

puntos (m, n + 1) y (m + 1, n) se encuentran vacíos, quitamos <strong>la</strong> piedra<br />

de (m, n) y ponemos una en (m, n + 1) y otra en (m + 1, n).<br />

¿Es posible <strong>que</strong> luego de una cantidad finita de pasos <strong>lo</strong>gremos dejar vacíos<br />

<strong>lo</strong>s puntos (0, 0), (0, 1) y (1, 0)?<br />

Lo primero <strong>que</strong> uno piensa es <strong>que</strong> debería poderse <strong>lo</strong>grar el objetivo. Uno se ve<br />

tentado a creer <strong>que</strong> podrá ir llevando cada piedra suficientemente lejos como <strong>para</strong><br />

<strong>que</strong> no moleste <strong>la</strong> retirada del resto. Luego de experimentar un poco caemos en <strong>que</strong><br />

no es tan simple. Cada vez tenemos más piedras y éstas cada vez van limitando más<br />

<strong>lo</strong>s movimientos del resto. De alguna forma, <strong>la</strong>s piedras más cercanas a (0, 0) son<br />

más “pesadas“ <strong>que</strong> <strong>la</strong>s más alejadas ya <strong>que</strong> les cuesta más moverse. Veamos como <strong>lo</strong>s<br />

invariantes nos ayudan a formalizar esto y demostrar <strong>que</strong> es imposible limpiar <strong>lo</strong>s 3<br />

puntos (0, 0), (0, 1) y (1, 0).<br />

1<br />

El peso de una piedra en el punto (m, n) será . Nuestro invariante es <strong>la</strong> suma de<br />

2 m+n<br />

<strong>lo</strong>s pesos de todas <strong>la</strong>s piedras, <strong>que</strong> l<strong>la</strong>maremos peso total. Notemos <strong>que</strong> efectivamente<br />

esta cantidad no varía en cada paso pues:<br />

1<br />

2 m+n = 1<br />

2 m+n+1 + 1<br />

2 m+1+n<br />

Inicialmente el peso total es 1 + 1 + 1 = 2. Como en cada punto a <strong>lo</strong> sumo<br />

2 2<br />

puede haber una piedra entonces es imposible llegar a una situación con <strong>lo</strong>s puntos<br />

(0, 0), (0, 1), (1, 0) vacíos ya <strong>que</strong> en ese caso el peso total sería < 2 pues:<br />

18


1. Las piedras en <strong>la</strong> primera columna a <strong>lo</strong> sumo suman 2 1 4 + 1 8 + 1<br />

16 + .... = 1 2<br />

2. Las piedras en <strong>la</strong> segunda columna a <strong>lo</strong> sumo suman 1 4 + 1 8 + 1<br />

16 + .... = 1 2<br />

3. Las piedras en <strong>la</strong> tercera columna a <strong>lo</strong> sumo suman 1 4 + 1 8 + 1<br />

16 + .... = 1 2<br />

4. Las piedras en <strong>la</strong> cuarta columna a <strong>lo</strong> sumo suman 1 8 + 1<br />

16 + 1<br />

32 + .... = 1 4<br />

5. Las piedras en <strong>la</strong> quinta columna a <strong>lo</strong> sumo suman 1<br />

16 + 1<br />

32 + 1<br />

64 + .... = 1 8<br />

6. .......<br />

Es decir <strong>que</strong> entre todas <strong>la</strong>s piedras a <strong>lo</strong> sumo suman<br />

1<br />

2 + 1 2 + 1 2 + 1 4 + 1 8 + 1 16 + .... = 1 + 1 = 2<br />

Pero <strong>para</strong> darse <strong>la</strong> igualdad deberíamos usar TODOS <strong>lo</strong>s puntos por <strong>lo</strong> <strong>que</strong> necesitaríamos<br />

infinitas piedras e infinitos turnos. ¡Absurdo! De donde no es posible dejar<br />

vacíos <strong>lo</strong>s puntos (0, 0), (0, 1) y (1, 0). ‡<br />

Algunos problemas <strong>para</strong> pensar<br />

1. Tenemos el mismo problema <strong>que</strong> antes pero con só<strong>lo</strong> 1 piedra en <strong>la</strong> casil<strong>la</strong> (0, 0).<br />

El objetivo es dejar vacías <strong>la</strong>s casil<strong>la</strong>s (0, 0), (0, 1), (1, 0), (0, 2), (1, 1), (2, 0). ¿Es<br />

posible <strong>lo</strong>grar<strong>lo</strong>? (Ésta es <strong>la</strong> segunda parte del problema original, <strong>la</strong> primera era<br />

con <strong>la</strong>s 6 casil<strong>la</strong>s ocupadas)<br />

2. (Moscú 1995) Tenemos 4 triángu<strong>lo</strong>s rectángu<strong>lo</strong>s iguales. En cada turno, podemos<br />

elegir cualquier triángu<strong>lo</strong> de <strong>lo</strong>s <strong>que</strong> tenemos y cortar<strong>lo</strong> en 2 por <strong>la</strong> altura<br />

del triángu<strong>lo</strong> rectángu<strong>lo</strong> (notar <strong>que</strong> <strong>que</strong>dan formados 2 triángu<strong>lo</strong>s rectángu<strong>lo</strong>s).<br />

Probar <strong>que</strong> siempre vamos a tener 2 triángu<strong>lo</strong>s iguales.<br />

3. Se tiene una tira infinita de casilleros con algunas piedras distribuidas entre el<strong>lo</strong>s<br />

(puede haber más de una piedra por casillero). En cada turno se pueden quitar<br />

2 piedras del mismo casillero y agregar una piedra al casillero inmediatamente<br />

a <strong>la</strong> derecha. Probar <strong>que</strong> sin importar como procedamos, llega un momento en<br />

<strong>que</strong> no podemos seguir moviendo piedras y <strong>la</strong> distribución final es independiente<br />

de nuestras elecciones.<br />

2 Para sumas de este tipo consultar el apéndice “Sumas infinitas“<br />

19


2.5. Fichas<br />

A veces resulta necesario llevar a cabo un procedimiento complejo <strong>para</strong> e<strong>la</strong>borar<br />

un invariante, pero el so<strong>lo</strong> hecho de proponerse <strong>la</strong> construcción del mismo puede llevar<br />

a <strong>que</strong> <strong>lo</strong> hallemos.<br />

Lo <strong>que</strong> sigue es un caso particu<strong>la</strong>r de un problema de <strong>la</strong> Olimpíada Rusa de 1997.<br />

El problema gustó tanto <strong>que</strong> fue votado como “El mejor Problema del año“ por <strong>lo</strong>s<br />

participantes.<br />

Consideremos una tira infinita de casilleros numerados con <strong>lo</strong>s números<br />

enteros. En algunos casilleros hay fichas de forma <strong>que</strong> <strong>la</strong> cantidad total de<br />

fichas es finita (puede haber casilleros con 2 o más fichas). En cada paso<br />

se pueden realizar 2 tipos de jugadas<br />

1. Si hay 2 casil<strong>la</strong>s no vacías seguidas, n y n + 1, se saca una de cada<br />

una y se pone una ficha en <strong>la</strong> casil<strong>la</strong> n + 2.<br />

2. Si hay al menos 2 fichas en <strong>la</strong> casil<strong>la</strong> n, se retiran <strong>la</strong>s 2 fichas y se<br />

pone una en <strong>la</strong> casil<strong>la</strong> n + 1 y otra en <strong>la</strong> casil<strong>la</strong> n − 2.<br />

Demostrar <strong>que</strong> toda secuencia de movidas conduce irremediablemente a<br />

una posición desde <strong>la</strong> cual es imposible realizar nuevas movidas y sin<br />

importar <strong>la</strong>s jugadas <strong>que</strong> se hicieron siempre se llega a <strong>la</strong> misma posición.<br />

Lo primero <strong>que</strong> haremos en este problema es buscar un invariante. La idea es<br />

escribir en cada casil<strong>la</strong> un número <strong>que</strong> representará el peso de cada ficha en esa casil<strong>la</strong><br />

de forma tal <strong>que</strong> <strong>la</strong> suma de <strong>lo</strong>s pesos de todas <strong>la</strong>s fichas (el “peso total“) no varíe al<br />

realizar <strong>la</strong>s operaciones permitidas. Veamos entonces cómo escribir <strong>lo</strong>s números.<br />

Si en <strong>la</strong> “n−ésima“ casil<strong>la</strong> escribimos el número a n (notar <strong>que</strong> n puede ser negativo)<br />

y cualquier operación deja invariante el peso total, entonces en cada paso <strong>la</strong> suma de<br />

<strong>lo</strong>s pesos de <strong>la</strong>s fichas <strong>que</strong> retiramos debe ser <strong>lo</strong> mismo <strong>que</strong> <strong>la</strong> suma de <strong>lo</strong>s pesos de<br />

<strong>la</strong>s fichas <strong>que</strong> agregamos. Esto se traduce en <strong>que</strong> <strong>para</strong> cada n:<br />

1. a n+2 = a n+1 + a n<br />

2. 2a n = a n+1 + a n−2<br />

Cuando se tienen sucesiones con este tipo de re<strong>la</strong>ciones 3 es común poner a n = α n<br />

<strong>para</strong> todo n y <strong>la</strong>s condiciones se transforman en:<br />

3 Mirar el apéndice “Sucesiones recurrentes“<br />

20


1. α n+2 = α n+1 + α n<br />

2. 2α n = α n+1 + α n−2<br />

Sacando factor común α n en <strong>la</strong> primer ecuación y α n−2 en <strong>la</strong> segunda llegamos a:<br />

1. α 2 = α + 1<br />

2. 2α 2 = α 3 + 1<br />

La ventaja de hacer el reemp<strong>la</strong>zo anterior es <strong>que</strong> de esta forma <strong>la</strong>s condiciones ya<br />

no dependen de n. Antes teníamos infinitas condiciones y ahora so<strong>la</strong>mente dos. No es<br />

difícil ver <strong>que</strong> α = 1+√ 5, <strong>que</strong> es <strong>la</strong> raíz positiva de x 2 − x − 1 = 0, satisface ambas.<br />

2<br />

Pongamos entonces α = 1+√ 5, escribamos <strong>para</strong> cada n el número α n en <strong>la</strong> n−ésima<br />

2<br />

casil<strong>la</strong> y decretemos <strong>que</strong> el peso de una ficha es el número escrito en <strong>la</strong> casil<strong>la</strong> <strong>que</strong><br />

ocupa. Como <strong>para</strong> todo n, α n+2 = α n+1 + α n y 2α n = α n+1 + α n−2 entonces en cada<br />

paso, sin importar qué operación realicemos, <strong>la</strong> suma de <strong>lo</strong>s pesos de <strong>la</strong>s fichas no<br />

varía. ¡Hemos encontrado el invariante!<br />

Probemos <strong>que</strong> toda configuración inicial T con peso total w(T ) conduce a una<br />

posición en donde no se pueden realizar más movimientos. Como α > 1, debe existir<br />

k 0 tal <strong>que</strong> α k > w(T ) <strong>para</strong> todo k ≥ k 0 . Al ser todos <strong>lo</strong>s pesos positivos nunca<br />

podremos poner una ficha en <strong>la</strong> casil<strong>la</strong> k 0 o a su derecha ya <strong>que</strong> ésta so<strong>la</strong> superaría el<br />

peso total. Podemos concluir entonces <strong>que</strong> hay una ficha <strong>que</strong> llegó más lejos hacia <strong>la</strong><br />

derecha <strong>que</strong> cualquier otra. Al hacer<strong>lo</strong>, esta ficha debe <strong>que</strong>darse quieta por el resto del<br />

juego pues si se mueve tendríamos <strong>que</strong> poner una ficha a su derecha <strong>lo</strong> <strong>que</strong> contradice<br />

su elección. C<strong>la</strong>vemos esta ficha y repitamos el argumento con el resto (haciendo de<br />

cuenta <strong>que</strong> <strong>la</strong> ficha c<strong>la</strong>vada no existe). Continuando de esta manera, como nunca se<br />

crean fichas, a <strong>lo</strong> sumo se descartan, tarde o temprano todas <strong>la</strong>s fichas deben estar<br />

c<strong>la</strong>vadas y no se podrán mover más fichas.<br />

Ahora debemos probar <strong>que</strong> sin importar el orden en <strong>que</strong> se hicieron <strong>la</strong>s jugadas<br />

siempre se llega a <strong>la</strong> misma posición. En caso contrario tendríamos 2 distribuciones<br />

distintas de fichas T i y T j , sin 2 en <strong>la</strong> misma casil<strong>la</strong> ni 2 en casil<strong>la</strong>s consecutivas, tales<br />

<strong>que</strong> w(T i ) = w(T j ). Veamos <strong>que</strong> esto es imposible, <strong>para</strong> el<strong>lo</strong> podemos suponer <strong>que</strong><br />

ninguna ficha de T i está en <strong>la</strong> misma casil<strong>la</strong> <strong>que</strong> una de T j , pues en ese caso quitamos<br />

esas fichas y tenemos un contraejemp<strong>lo</strong> más chico. Si <strong>la</strong>s fichas de T i están en <strong>la</strong>s<br />

casil<strong>la</strong>s i 1 < i 2 < ... < i n (con i k−1 ≤ i k − 2) y <strong>la</strong>s de T j en j 1 < j 2 < ... < j m (con<br />

j k−1 ≤ j k − 2) luego 4 :<br />

4 Consultar el apéndice “Sumas infinitas“<br />

w(T i ) = α in + α i n−1<br />

+ .... + α i 1<br />

21


w(T i ) < α in + α in−2 + α in−4 + ....<br />

w(T i ) < α in (1 + α −2 + α −4 + ...)<br />

w(T i ) < α in 1<br />

1 − α −2 = αin α 2<br />

(α 2 − 1)<br />

α2 in<br />

w(T i ) < α<br />

α = αin+1<br />

Luego α in+1 > w(T i ) = w(T j ) ≥ α jm , es decir <strong>que</strong> i n + 1 > j m o <strong>lo</strong> <strong>que</strong> es <strong>lo</strong> mismo<br />

<strong>que</strong> i n ≥ j m , pero de forma aná<strong>lo</strong>ga j m ≥ i n y podemos concluir <strong>que</strong> i n = j m . Es decir<br />

<strong>que</strong> T i tiene una ficha en <strong>la</strong> misma casil<strong>la</strong> <strong>que</strong> T j tiene otra, <strong>lo</strong> <strong>que</strong> contradice nuestra<br />

suposición. Entonces cualesquiera 2 distribuciones desde <strong>la</strong>s <strong>que</strong> no se puedan realizar<br />

movidas con el mismo peso total deben ser iguales.<br />

Hemos probado entonces <strong>que</strong> no importa qué <strong>la</strong>s elecciones <strong>que</strong> tomemos, siempre<br />

se llega a una configuración desde donde no es posible mover más fichas y esta<br />

configuración siempre es <strong>la</strong> misma. ‡<br />

En el problema anterior <strong>que</strong>ríamos asignar números de forma tal <strong>que</strong> determinada<br />

cuenta sencil<strong>la</strong> permaneciera invariante a <strong>la</strong>s operaciones del problema; <strong>lo</strong>s <strong>que</strong> no<br />

resultarían ser tan sencil<strong>lo</strong>s son <strong>lo</strong>s números <strong>que</strong> debíamos encontrar <strong>para</strong> <strong>que</strong> se<br />

cumpliera eso, pero el hecho es <strong>que</strong> al p<strong>la</strong>ntear <strong>la</strong>s condiciones estos aparecieron de<br />

forma natural.<br />

Algunos problemas <strong>para</strong> pensar<br />

1. ¿Qué pasa si en <strong>la</strong> solución anterior en vez de poner a n = α n ponemos a n = F n<br />

donde F n es el n−ésimo término de <strong>la</strong> sucesión de Fibonacci definida recursivamente<br />

por F 0 = 0, F 1 = 1 y F n+2 = F n+1 + F n ?<br />

2. (Rusia 1997) Sea k ≥ 2 un número natural. Consideremos una tira infinita de<br />

casilleros numerados con <strong>lo</strong>s números enteros. En algunos casilleros hay fichas<br />

de forma <strong>que</strong> <strong>la</strong> cantidad total de fichas es finita (puede haber casilleros con 2<br />

o más fichas). En cada paso se pueden realizar 2 tipos de jugadas<br />

a) Si hay k casil<strong>la</strong>s no vacías seguidas, n, n + 1, ..., n + k − 1, se saca una ficha<br />

de cada una y se pone una ficha en <strong>la</strong> casil<strong>la</strong> n + k.<br />

b) Si hay al menos 2 fichas en <strong>la</strong> casil<strong>la</strong> n, se retiran <strong>la</strong>s 2 fichas y se pone<br />

una en <strong>la</strong> casil<strong>la</strong> n + 1 y otra en <strong>la</strong> casil<strong>la</strong> n − k.<br />

Demostrar <strong>que</strong> toda secuencia de movidas conduce irremediablemente a una<br />

posición desde <strong>la</strong> cual es imposible realizar nuevas movidas y sin importar <strong>la</strong>s<br />

jugadas <strong>que</strong> se hicieron siempre se llega a <strong>la</strong> misma posición.<br />

22


3. Sea {F n } <strong>la</strong> sucesión de Fibonacci definida recursivamente por F 0 = 0, F 1 = 1<br />

y F n+2 = F n+1 + F n . Probar <strong>que</strong> todo numero natural n > 0 tiene una única<br />

expresión de <strong>la</strong> forma<br />

n = F i1 + F i2 + ..... + F ik<br />

donde i 1 ≥ 2 y i j+1 ≥ i j + 2 <strong>para</strong> todo j.<br />

4. (Apmo 2006) Probar <strong>que</strong> todo entero positivo se puede escribir como <strong>la</strong> suma<br />

finita de algunas potencias distintas de el número de oro. Es decir, si α =<br />

1+ √ 5<br />

2<br />

entonces <strong>para</strong> todo n entero positivo, existen i 1 , ..., i k enteros distintos no<br />

necesariamente positivos tales <strong>que</strong><br />

n = α i 1<br />

+ ........ + α i k<br />

5. (Imo 1993) Construir una función f : N → N tal <strong>que</strong> f(1) = 2 y <strong>para</strong> todo<br />

n ∈ N<br />

f(f(n)) = f(n) + n<br />

f(n) < f(n + 1)<br />

6. Se tiene una pi<strong>la</strong> de n fichas <strong>que</strong> forman una columna vertical. Esta configuración<br />

se modifica de acuerdo a <strong>la</strong>s siguientes reg<strong>la</strong>s. Una ficha se puede mover si<br />

está en el tope de una columna <strong>que</strong> contiene por <strong>lo</strong> menos dos fichas más <strong>que</strong><br />

<strong>la</strong> columna <strong>que</strong> está inmediatamente a <strong>la</strong> derecha (si no hay columnas a <strong>la</strong><br />

derecha, considerar <strong>que</strong> hay columnas con 0 fichas). En cada etapa, se elige<br />

una de <strong>la</strong>s fichas <strong>que</strong> se pueden mover (si es <strong>que</strong> hay alguna) y se co<strong>lo</strong>ca en<br />

el tope de <strong>la</strong> columna <strong>que</strong> está a su derecha. Si no se puede mover ninguna<br />

ficha <strong>la</strong> configuración se l<strong>la</strong>ma final. Para cada n, demostrar <strong>que</strong> no importa<br />

qué elecciones se tomen en cada etapa, <strong>la</strong> configuración final <strong>que</strong> se obtiene es<br />

única. Describir dicha configuración en términos de n.<br />

23


2.6. Un poco de Geometría Proyectiva<br />

Dado un punto o y una recta L <strong>que</strong> no pasa por o, <strong>la</strong> proyección en <strong>la</strong> recta L de<br />

un punto a desde o es el punto a ′ <strong>que</strong> se obtiene como <strong>la</strong> intersección de ao y L 5 .<br />

Inicialmente tenemos 4 puntos alineados a 0 , b 0 , c 0 , d 0 (en ese orden) tales<br />

<strong>que</strong> a 0 b 0 = b 0 c 0 = c 0 d 0 = 1. En cada turno podemos elegir una recta y<br />

proyectar <strong>lo</strong>s 4 puntos en <strong>la</strong> recta desde cualquier punto, luego borramos<br />

<strong>lo</strong>s 4 puntos <strong>que</strong> proyectamos y nos <strong>que</strong>damos con <strong>lo</strong>s nuevos. ¿Es posible<br />

<strong>que</strong> luego de varios turnos obtengamos 4 puntos alineados a 1 , b 1 , c 1 , d 1 (en<br />

ese orden) tales <strong>que</strong> a 1 b 1 = 2b 1 c 1 = c 1 d 1 = 2?<br />

Supongamos <strong>que</strong> es posible y lleguemos a un absurdo. Para el<strong>lo</strong> vamos a demostrar<br />

<strong>que</strong> <strong>la</strong> razón doble de <strong>lo</strong>s 4 puntos permanece invariante. Si tenemos 4 puntos a, b, c, d<br />

su razón doble se denota por {ac : bd} y se define como:<br />

{ac : bd} = ab<br />

cb /ad ab · cd<br />

=<br />

cd cb · ad<br />

Consideremos ahora un punto o y proyectemos 4 puntos a, b, c, d de una recta L<br />

en otra L ′ , obteniendo a ′ , b ′ , c ′ , d ′ . Debemos probar <strong>que</strong> {ac : bd} = {a ′ c ′ : b ′ d ′ }.<br />

Sea p <strong>la</strong> intersección de L y L ′ . Usando el Teorema de Menea<strong>lo</strong> en <strong>lo</strong>s triángu<strong>lo</strong>s<br />

apa ′ y cpc ′ con <strong>la</strong> recta bb ′ o obtenemos:<br />

pb · a ′ b ′<br />

= a′ o<br />

ab · pb ′ ao<br />

pb · c ′ b ′<br />

= c′ o<br />

bc · pb ′ co<br />

(1)<br />

(2)<br />

de L.<br />

5 Si ao//L decimos <strong>que</strong> a se fue al punto del infinito de L o <strong>que</strong> se fue al infinito en <strong>la</strong> dirección<br />

24


Ahora volvamos a usar el Teorema de Mene<strong>la</strong>o en <strong>lo</strong>s triángu<strong>lo</strong>s apa ′ y cpc ′ pero<br />

con <strong>la</strong> recta dd ′ o.<br />

pd · a ′ d ′<br />

= a′ o<br />

ad · pd ′ ao<br />

pd · c ′ d ′<br />

= c′ o<br />

dc · pd ′ co<br />

(3)<br />

(4)<br />

Luego<br />

De donde<br />

pb · a ′ b ′ pd · c ′ d ′<br />

= pb · c′ b ′ pd · a ′ d ′<br />

ab · pb ′ dc · pd ′ bc · pb ′ ad · pd ′<br />

a ′ b ′ · c ′ d ′<br />

= c′ b ′ · a ′ d ′<br />

ab · dc bc · ad<br />

a ′ b ′ · c ′ d ′ ab · dc<br />

=<br />

c ′ b ′ · a ′ d ′ bc · ad<br />

{ac : bd} = {a ′ c ′ : b ′ d ′ }<br />

Por <strong>lo</strong> recién visto, si fuera posible empezar con a 0 , b 0 , c 0 , d 0<br />

a 1 , b 1 , c 1 , d 1 entonces:<br />

y terminar con<br />

Pero esto es absurdo pues<br />

{a 0 c 0 : b 0 d 0 } = {a 1 c 1 : b 1 d 1 }<br />

{a 0 c 0 : b 0 d 0 } = a 0b 0 d 0 c 0<br />

b 0 c 0 a 0 d 0<br />

= 1 · 1<br />

1 · 3 = 1 3<br />

{a 1 c 1 : b 1 d 1 } = a 1b 1 d 1 c 1<br />

b 1 c 1 a 1 d 1<br />

= 2 · 2<br />

1 · 5 = 4 5<br />

Entonces no es posible <strong>lo</strong> <strong>que</strong> pide el enunciado. ‡<br />

Lo anterior es un ejemp<strong>lo</strong> de algo muy usado en geometría; “Las Transformaciones“.<br />

Veamos otro ejemp<strong>lo</strong>. Imaginemos un punto o y 2 p<strong>la</strong>nos π y π ′ . La proyección de π<br />

sobre π ′ desde o es a cada punto p en π mandar<strong>lo</strong> a <strong>la</strong> intersección p ′ de op con π ′ 6 .<br />

¿Qué preserva <strong>la</strong> proyección?<br />

6 Si op//π ′ entonces p ′ es el punto del infinito de π ′ en <strong>la</strong> dirección de L.<br />

25


La misma cuenta <strong>que</strong> hicimos en <strong>la</strong> solución anterior demuestra <strong>que</strong> <strong>la</strong> razón<br />

doble de puntos alineados es invariante en este tipo de proyección; pero hay otra<br />

estructura más simple <strong>que</strong> se preserva. Notemos <strong>que</strong> si a, b, c están en π y a ′ , b ′ , c ′ son<br />

sus proyecciones en π ′ entonces a, b, c están alineados si y só<strong>lo</strong> si a ′ , b ′ , c ′ <strong>lo</strong> están. Dicho<br />

de otra forma, <strong>la</strong> proyección manda “rectas en rectas“. Más aún si P, Q, R son 3 rectas<br />

en π y P ′ , Q ′ , R ′ son sus proyecciones sobre π ′ entonces P, Q, R son concurrentes si y<br />

só<strong>lo</strong> si P ′ , Q ′ , R ′ <strong>lo</strong> son 7 .<br />

De ahora en ade<strong>la</strong>nte, cuando tengamos un objeto T en π su proyección T ′ es el<br />

conjunto de puntos de π ′ <strong>que</strong> son <strong>la</strong>s proyecciones de puntos de T . La proyección de<br />

un círcu<strong>lo</strong> no es necesariamente un círcu<strong>lo</strong>. Lo <strong>que</strong> ocurre es <strong>que</strong> no estamos mirando<br />

<strong>la</strong> propiedad correcta, el círcu<strong>lo</strong> es miembro de una familia más grande de curvas: “<strong>la</strong>s<br />

cónicas“ 8 . Los 4 miembros de esta familia son <strong>lo</strong>s círcu<strong>lo</strong>s, <strong>la</strong>s parábo<strong>la</strong>s, <strong>la</strong>s elipses<br />

y <strong>la</strong>s hipérbo<strong>la</strong>s. La proyección de una cónica es una cónica. Es decir “ser un círcu<strong>lo</strong>“<br />

no es invariante por proyección, pero sí <strong>lo</strong> es “ser una cónica“.<br />

Otra propiedad invariante por proyección es “ser tangente“ a una cónica. Es decir<br />

si tenemos una recta L y una cónica ω y L ′ y ω ′ son sus proyecciones entonces L<br />

es tangente a ω si y só<strong>lo</strong> si L ′ es tangente a ω ′ . Dado un punto p en el exterior de<br />

una cónica, ¿será posible trazar <strong>la</strong>s tangentes a <strong>la</strong> cónica usando só<strong>lo</strong> una reg<strong>la</strong>? Por<br />

ejemp<strong>lo</strong> <strong>para</strong> un círcu<strong>lo</strong> podemos proceder de <strong>la</strong> siguiente forma:<br />

Dado un círcu<strong>lo</strong> ω y un punto p en su exterior tomemos 2 puntos arbitrarios<br />

a y b en ω y tracemos <strong>la</strong>s rectas pa y pb <strong>que</strong> vuelven a cortar a ω<br />

en c y d respectivamente. Tracemos ahora <strong>la</strong>s rectas ab y cd <strong>que</strong> se cortan<br />

en q y ad y bc <strong>que</strong> se cortan en r. Por último tracemos qr <strong>que</strong> corta a ω<br />

en s y t. Luego ps y pt son <strong>la</strong>s tangentes a ω por p.<br />

7 ¡Cuidado! si P y Q son 2 rectas <strong>para</strong>le<strong>la</strong>s, P ′ y Q ′ no necesariamente <strong>lo</strong> son.<br />

8 Cada cónica ω se puede pensar como <strong>la</strong> intersección de un cono con un p<strong>la</strong>no.<br />

26


Ahora, como <strong>la</strong> construcción entera es invariante por proyección, el mismo procedimiento<br />

funciona <strong>para</strong> trazar <strong>la</strong>s tangentes a una cónica. Es decir, supongamos <strong>que</strong><br />

realizamos <strong>la</strong> construcción anterior <strong>para</strong> una cónica ω y <strong>que</strong>remos probar <strong>que</strong> pr y qr<br />

resultan tangentes a ω. Proyectemos el dibujo a una p<strong>la</strong>no π ′ de forma <strong>que</strong> <strong>la</strong> proyección<br />

ω ′ de ω resulte un círcu<strong>lo</strong> (¿Se les ocurre cómo hacer esto?). Ahora tenemos un<br />

punto p ′ y 2 rectas por él <strong>que</strong> cortan a ω ′ en a ′ y c ′ <strong>la</strong> primera y b ′ y d ′ <strong>la</strong> segunda.<br />

La intersección de a ′ b ′ y c ′ d ′ es q ′ y <strong>la</strong> intersección de a ′ d ′ y b ′ c ′ es r ′ . Por último, q ′ r ′<br />

corta a ω ′ en s ′ y t ′ . Como esto no es otra cosa <strong>que</strong> <strong>la</strong> construcción anterior aplicada<br />

a ω ′ y p ′ , resulta <strong>que</strong> p ′ s ′ y p ′ t ′ son tangentes a ω ′ , de donde proyectando de vuelta,<br />

obtenemos <strong>que</strong> ps y pt son tangentes a ω.<br />

¿Será posible usando só<strong>lo</strong> una reg<strong>la</strong> marcar el centro de un círcu<strong>lo</strong>? Más<br />

precisamente, <strong>lo</strong> <strong>que</strong> preguntamos es si existe un procedimiento <strong>que</strong> dado<br />

cualquier círcu<strong>lo</strong> nos permita encontrar su centro usando só<strong>lo</strong> una reg<strong>la</strong><br />

(como el anterior <strong>para</strong> trazar <strong>la</strong>s tangentes a un círcu<strong>lo</strong> por un punto en<br />

su exterior). Con <strong>la</strong> reg<strong>la</strong> <strong>lo</strong> <strong>que</strong> podemos hacer es <strong>lo</strong> siguiente<br />

1. Trazar <strong>la</strong> recta <strong>que</strong> une 2 puntos ya marcados.<br />

2. Marcar <strong>la</strong> intersección de 2 rectas ya dibujadas.<br />

3. Trazar una recta arbitraria por un punto dado.<br />

4. Marcar un punto arbitrario sobre el círcu<strong>lo</strong> original o sobre una recta<br />

ya dibujada.<br />

No, no es posible. Supongamos <strong>que</strong> sí, tenemos entonces un procedimiento <strong>que</strong><br />

<strong>para</strong> cualquier círcu<strong>lo</strong> nos permite encontrar su centro usando só<strong>lo</strong> una reg<strong>la</strong> como en<br />

1,2,3 o 4.<br />

27


Imaginemos 2 p<strong>la</strong>nos π y π ′ y 2 círcu<strong>lo</strong>s ω y ω ′ de forma <strong>que</strong> ω está en π y ω ′<br />

es <strong>la</strong> proyección de ω sobre π ′ . Apli<strong>que</strong>mos nuestro procedimiento <strong>para</strong> determinar el<br />

centro de ω de forma <strong>que</strong> cada cosa <strong>que</strong> vamos dibujando en π <strong>la</strong> vamos proyectando<br />

a π ′ . ¿Qué podemos decir de <strong>lo</strong> dibujado en π ′ ? Resulta <strong>que</strong> si alguien viene y observa<br />

π ′ sin <strong>saber</strong> <strong>lo</strong> <strong>que</strong> estamos haciendo en π creerá <strong>que</strong> estamos aplicando nuestro<br />

procedimiento <strong>para</strong> determinar el centro de ω ′ pues al proyectar <strong>lo</strong> <strong>que</strong> hacemos en π<br />

es como si estuviéramos haciéndo<strong>lo</strong> en π ′ . Por ejemp<strong>lo</strong>, si tenemos a y b en π, trazamos<br />

<strong>la</strong> recta L <strong>que</strong> <strong>lo</strong>s une y <strong>la</strong> proyectamos obteniendo L ′ , es como si directamente<br />

hubiéramos trazado <strong>la</strong> recta <strong>que</strong> une a ′ con b ′ . Si marcamos <strong>la</strong> intersección p de 2<br />

rectas L 1 y L 2 y <strong>la</strong> proyectamos a π ′ obteniendo p ′ es como si hubiéramos marcado<br />

<strong>la</strong> intersección de L ′ 1 y L ′ 2.<br />

Como nuestra construcción sirve <strong>para</strong> cualquier círcu<strong>lo</strong> independientemente de<br />

nuestras elecciones (<strong>que</strong> vienen de usar <strong>la</strong> reg<strong>la</strong> en cosas como en 3 y 4) entonces<br />

cuando encontremos el centro de ω y <strong>lo</strong> proyectemos hasta π ′ , éste será el centro de<br />

ω, pues el dibujo en π ′ es nuestro procedimiento aplicado <strong>para</strong> determinar el centro<br />

de ω ′ . Dicho de otra forma.. ¡Nuestro procedimiento es invariante por proyección!<br />

El problema es <strong>que</strong> si bien nuestro procedimiento es invariante por proyección, no<br />

<strong>lo</strong> es <strong>la</strong> propiedad “ser el centro de un círcu<strong>lo</strong>“. En realidad, como dijimos antes ni<br />

siquiera “ser un círcu<strong>lo</strong>“ <strong>lo</strong> es, de forma <strong>que</strong> debemos tener un poco de cuidado.<br />

Para terminar el problema <strong>lo</strong> <strong>que</strong> hacemos es elegir <strong>lo</strong>s p<strong>la</strong>nos π y π ′ y <strong>lo</strong>s círcu<strong>lo</strong>s<br />

ω y ω ′ de forma <strong>que</strong> <strong>la</strong> proyección del centro de ω no sea el centro de ω ′ 9 , <strong>lo</strong> <strong>que</strong><br />

contradice nuestra conclusión anterior. Entonces no puede existir un procedimiento<br />

<strong>que</strong> nos permita encontrar el centro de un círcu<strong>lo</strong> usando só<strong>lo</strong> una reg<strong>la</strong>. ‡<br />

La demostración anterior merece algunos comentarios. En primer lugar, el problema<br />

anterior no es un problema común. Una cosa es encontrar un procedimiento<br />

<strong>para</strong> hacer algo pero otra dramáticamente distinta es probar <strong>que</strong> tal procedimiento<br />

no puede existir. Ejemp<strong>lo</strong>s de esta situación son <strong>la</strong> cuadratura del círcu<strong>lo</strong>, <strong>la</strong> construcción<br />

de algunos polígonos regu<strong>la</strong>res con reg<strong>la</strong> y compás o <strong>la</strong> resolución de ecuaciones<br />

de grado ≥ 5. Probar <strong>que</strong> no existe ninguna fórmu<strong>la</strong> <strong>para</strong> encontrar <strong>la</strong>s raíces de<br />

una ecuación de grado 5 precisó (y motivó) del desarrol<strong>lo</strong> de uno de <strong>lo</strong>s trabajos más<br />

profundos, bel<strong>lo</strong>s e importantes de <strong>la</strong> matemática: “La Teoría de Ga<strong>lo</strong>is“. En segundo<br />

lugar, hay algo en <strong>la</strong> demostración anterior <strong>que</strong> <strong>la</strong> hace “especial“, el problema hab<strong>la</strong>ba<br />

de algo <strong>que</strong> ocurre en el p<strong>la</strong>no pero <strong>la</strong> demostración es en 3 dimensiones. Otro ejemp<strong>lo</strong><br />

de cuan importante es no encasil<strong>la</strong>r el pensamiento de uno, de no limitar<strong>lo</strong>, de dejar<br />

vo<strong>la</strong>r <strong>la</strong> imaginación.<br />

9 Mirar el problema 11<br />

28


Algunos problemas <strong>para</strong> pensar<br />

1. Dado un punto o, un ángu<strong>lo</strong> α y una constante k consideremos <strong>la</strong> siguiente<br />

transformación del p<strong>la</strong>no en sí mismo. A cada punto p <strong>lo</strong> mandamos a p ′ de<br />

forma <strong>que</strong> pop ′ = α y po = k (a mí me gusta pensar<strong>la</strong> como <strong>que</strong> tenemos un<br />

p ′ o<br />

triángu<strong>lo</strong> T y cada punto p <strong>lo</strong> mandamos a p ′ de forma <strong>que</strong> pop ′ sea semejante a<br />

T ). Probar <strong>que</strong> preserva colinealidad, ángu<strong>lo</strong>s entre rectas y razón de segmentos.<br />

2. Dado un punto o y una constante k, consideremos <strong>la</strong> siguiente transformación del<br />

p<strong>la</strong>no en sí mismo. A cada punto p mandémos<strong>lo</strong> a un punto p ′ en <strong>la</strong> semirrecta<br />

op tal <strong>que</strong> op×op ′ = k 2 . Probar <strong>que</strong> <strong>la</strong> transformación manda círcu<strong>lo</strong>s en círcu<strong>lo</strong>s<br />

(<strong>para</strong> este problema considerar <strong>la</strong>s rectas como círcu<strong>lo</strong>s de radio infinito). Probar<br />

además <strong>que</strong><br />

a ′ b ′ ab · k2<br />

=<br />

oa · ob<br />

Deducir <strong>que</strong> preserva razón doble.<br />

3. Dado un cuadrilátero convexo abcd probar <strong>que</strong> <strong>lo</strong>s 4 vértices están en una circunferencia<br />

o en una recta si y só<strong>lo</strong> si<br />

ab · cd + ad · bc = ac + bd<br />

4. Dado un triángu<strong>lo</strong> abc y un círcu<strong>lo</strong> ω <strong>que</strong> corta a ab en d y e, a bc en f y g y a<br />

ca en h e i sean p, q y r <strong>la</strong>s intersecciones de <strong>la</strong>s tangentes al círcu<strong>lo</strong> en d y e, f<br />

y g y en h e i. Probar <strong>que</strong> aq, br y cp son concurrentes.<br />

5. Dado un punto o en una circunferencia ω y una recta L definimos <strong>la</strong> proyección<br />

de ω en L como <strong>la</strong> transformación <strong>que</strong> a cada punto p en ω <strong>lo</strong> manda a p ′ en L<br />

<strong>que</strong> es <strong>la</strong> intersección de op con L. Probar <strong>que</strong> preserva razón doble.<br />

6. Una transformación de un círcu<strong>lo</strong> o una recta en otro círcu<strong>lo</strong> o recta se dice<br />

proyectiva si se obtiene como composición de proyecciones (por ejemp<strong>lo</strong> primero<br />

proyectamos a un círcu<strong>lo</strong>, luego a una recta (no necesariamente desde el mismo<br />

punto), luego a otra recta y así). Probar <strong>que</strong> <strong>la</strong>s transformaciones proyectivas<br />

preservan razón doble.<br />

7. Consideremos 2 p<strong>la</strong>nos π y π ′ y una recta L <strong>que</strong> no es <strong>para</strong>le<strong>la</strong> a ninguno de <strong>lo</strong>s 2<br />

p<strong>la</strong>nos. A cada punto p en π <strong>lo</strong> mandamos un punto p ′ en π ′ <strong>que</strong> es <strong>la</strong> intersección<br />

de una recta <strong>para</strong>le<strong>la</strong> a L por p con π ′ (es como si proyectáramos desde un punto<br />

en el infinito en <strong>la</strong> dirección de L). Probar <strong>que</strong> esta transformación preserva<br />

colinealidad, concurrencia, razón de segmentos <strong>para</strong>le<strong>lo</strong>s y razones de áreas.<br />

29


8. Probar <strong>que</strong> dado un triángu<strong>lo</strong> T y 3 puntos a, b y c en un p<strong>la</strong>no π podemos<br />

transformar<strong>lo</strong>s mediante una transformación como <strong>la</strong> del problema anterior en<br />

a ′ , b ′ , c ′ en un p<strong>la</strong>no π ′ de forma <strong>que</strong> a ′ b ′ c ′ sea semejante a T .<br />

9. Sea abcde un pentágono convexo y consideremos f = bc ∩ de, g = cd ∩ ea,<br />

h = de ∩ ab, i = ea ∩ bc y j = ab ∩ cd. Probar <strong>que</strong> si <strong>la</strong>s áreas de <strong>lo</strong>s triángu<strong>lo</strong>s<br />

ahi, bij, cjf, dfg, egh son todas iguales entonces af, bg, ch, di, ej son concurrentes.<br />

Sugerencia: Proyectar de forma <strong>que</strong> a ′ c ′ d ′ vaya a <strong>para</strong>r a <strong>lo</strong>s vértices de un<br />

pentágono regu<strong>la</strong>r.<br />

10. Probar <strong>que</strong> no es posible usando só<strong>lo</strong> una reg<strong>la</strong> trazar una <strong>para</strong>le<strong>la</strong> a un segmento<br />

dado.<br />

11. Supongamos <strong>que</strong> tenemos una esfera S y un p<strong>la</strong>no π ′ tangente a S en a. Sea<br />

o el punto de S diametralmente opuesto a a. Tomemos un p<strong>la</strong>no π <strong>que</strong> pase<br />

por el centro de S pero <strong>que</strong> no sea <strong>para</strong>le<strong>lo</strong> a π ′ y l<strong>la</strong>memos ω al círcu<strong>lo</strong> <strong>que</strong><br />

es <strong>la</strong> intersección de S con π. Por último tomemos p en π ′ de forma <strong>que</strong> op sea<br />

perpendicu<strong>la</strong>r a π. Probar <strong>que</strong> ω ′ , <strong>la</strong> proyección de ω a π ′ desde o, es un círcu<strong>lo</strong><br />

de centro p.<br />

30


2.7. Cartas<br />

Esta vez el problema es de <strong>la</strong> lista corta de <strong>la</strong> 46 IMO del 2005.<br />

Se tienen n cartas alineadas en una fi<strong>la</strong>, cada una de el<strong>la</strong>s con un <strong>la</strong>do<br />

b<strong>la</strong>nco y el otro negro, todas con su <strong>la</strong>do b<strong>la</strong>nco hacia arriba. En cada<br />

etapa, si es posible, se elige una carta con el <strong>la</strong>do b<strong>la</strong>nco hacia arriba (<strong>que</strong><br />

no está en el borde de <strong>la</strong> fi<strong>la</strong>), se quita y se dan vuelta <strong>la</strong> carta más próxima<br />

hacia <strong>la</strong> derecha y <strong>la</strong> carta más próxima hacia <strong>la</strong> izquierda. Demostrar <strong>que</strong><br />

se puede llegar a una situación en <strong>la</strong> <strong>que</strong> só<strong>lo</strong> <strong>que</strong>dan dos cartas si y só<strong>lo</strong><br />

si n − 1 no es divisible por 3.<br />

En primer lugar veamos <strong>que</strong> si n − 1 no es divisible por 3 entonces es posible dejar<br />

só<strong>lo</strong> 2 cartas. Notemos <strong>que</strong> si primero sacamos <strong>la</strong> segunda carta, luego <strong>la</strong> cuarta y por<br />

último <strong>la</strong> tercera entonces volvemos a <strong>la</strong> situación original pero con 3 cartas menos.<br />

BBBBB....... → NNBB....... → NBN....... → BB.......<br />

Luego, si n − 1 no es divisible por 3, repitiendo <strong>lo</strong> anterior vamos sacando de a<br />

3 cartas hasta llegar a BB o a BBB. Si llegamos a BB ya estamos y si llegamos a<br />

BBB entonces quitamos <strong>la</strong> del medio y estamos en NN; en ambos casos <strong>lo</strong>gramos<br />

dejar só<strong>lo</strong> 2 cartas, como <strong>que</strong>ríamos.<br />

Para ver <strong>que</strong> si n − 1 es divisible por 3 entonces no es posible dejar só<strong>lo</strong> 2 cartas<br />

vamos a usar un invariante. Esta vez va a ser necesario de una construcción auxiliar<br />

<strong>para</strong> entender<strong>lo</strong> mejor.<br />

Vamos a pensar cada carta como una orden. Dados 3 números ordenados, <strong>la</strong>s<br />

cartas negras intercambian <strong>lo</strong>s 2 últimos y <strong>la</strong>s b<strong>la</strong>ncas mueven cada número un lugar<br />

a <strong>la</strong> derecha (es decir <strong>que</strong> el primero toma el lugar del segundo, el segundo el del<br />

tercero y el tercero el del primero).<br />

N(xyz) = xzy<br />

B(xyz) = zxy<br />

Las órdenes se leen de derecha a izquierda, por ejemp<strong>lo</strong> BBN(123) = 321 pues<br />

N(123) = 132<br />

B(132) = 213<br />

B(213) = 321<br />

31


O en <strong>la</strong> notación <strong>que</strong> vamos a usar<br />

BBN(123) = BB(N(123)) = BB(132) = B(B(132)) = B(213) = 321<br />

No es difícil ver <strong>que</strong> tenemos <strong>la</strong>s siguientes re<strong>la</strong>ciones<br />

Por ejemp<strong>lo</strong><br />

BBB = NN (1)<br />

BBN = NB (2)<br />

NBB = BN (3)<br />

NBN = BB (4)<br />

NBN(xyz) = NB(xzy) = N(yxz) = yzx<br />

BB(xyz) = B(zxy) = yzx<br />

Además BBB(xyz) = NN(xyz) = xyz.<br />

Volviendo al problema, consideremos <strong>la</strong>s siguientes jugadas. Empezamos con 5<br />

cartas b<strong>la</strong>ncas; primero quitamos <strong>la</strong> segunda carta (de izquierda a derecha), luego <strong>la</strong><br />

cuarta y por último <strong>la</strong> tercera. La situación <strong>la</strong> podemos representar como sigue:<br />

BBBBB → NNBB → NBN → BB<br />

¿Qué tienen en común BBBBB, NNBB, NBN y BB? Para poder responder<br />

esta pregunta es <strong>que</strong> precisamos pensar a <strong>la</strong>s cartas como órdenes <strong>que</strong> se aplican a<br />

123. Las 4 secuencias tienen en común <strong>que</strong> pensadas como un conjunto de órdenes<br />

aplicadas a 123 dan <strong>lo</strong> mismo, es decir:<br />

BBBBB(123) = 231<br />

NNBB(123) = 231<br />

NBN(123) = 231<br />

BB(123) = 231<br />

Es c<strong>la</strong>ro ahora cuál será el invariante, hay <strong>que</strong> pensar a <strong>la</strong>s cartas como órdenes<br />

y aplicar<strong>la</strong>s (de derecha a izquierda) a 123. Para el<strong>lo</strong>, debemos verificar <strong>que</strong> en cada<br />

paso el resultado final no cambia. Supongamos por ejemp<strong>lo</strong> <strong>que</strong> sacamos una carta<br />

b<strong>la</strong>nca con 2 vecinas negras (<strong>lo</strong>s demás casos se deducen de <strong>la</strong> misma forma de <strong>la</strong>s<br />

re<strong>la</strong>ciones (1), (2) y (3)), entonces debemos ver <strong>que</strong><br />

...NBN...(123) = ...BB...(123)<br />

32


Pero de (4) sabemos <strong>que</strong> NBN = BB luego<br />

Como <strong>que</strong>ríamos.<br />

...NBN...(123) = ...NBN(...(123))<br />

...NBN...(123) = ...(NBN(...(123)))<br />

...NBN...(123) = ...(BB(...(123)))<br />

...NBN...(123) = ...BB(...(123))<br />

...NBN...(123) = ...BB...(123)<br />

Para terminar el problema, como BBB(123) = 123, so<strong>la</strong>mente hace falta notar<br />

<strong>que</strong> si n = 3k + 1 entonces<br />

BBBB......BBB(123) = BBBB......(123) = BBBB(123) = B(123) = 312<br />

De donde no es posible <strong>que</strong> <strong>que</strong>den 2 cartas pues<br />

BB(123) = B(312) = 231<br />

NB(123) = N(312) = 321<br />

BN(123) = B(132) = 213<br />

NN(123) = N(132) = 123<br />

es decir <strong>que</strong> ninguno coincide con 312. ‡<br />

La demostración anterior se puede hacer más gráfica pensando <strong>que</strong> 123 son <strong>lo</strong>s<br />

vértices de un triángu<strong>lo</strong> equilátero. Cada vez <strong>que</strong> tenemos una B <strong>lo</strong> rotamos 120 o en<br />

sentido antihorario y cada vez <strong>que</strong> tenemos una N <strong>lo</strong> reflejamos por <strong>la</strong> altura vertical<br />

(es decir <strong>que</strong> intercambiamos <strong>lo</strong>s 2 vértices de abajo).<br />

33


Completemos ahora <strong>la</strong> construcción del problema, así como definimos N(xyz) =<br />

xzy y B(xyz) = zxy podríamos haber definido otras órdenes...<br />

I(xyz) = xyz<br />

B 1 (xyz) = zxy<br />

B 2 (xyz) = yzx<br />

Ahora por ejemp<strong>lo</strong> B 1 N 1 N 2 (xyz) = yzx pues<br />

N 1 (xyz) = xzy<br />

N 2 (xyz) = zyx<br />

N 3 (xyz) = yxz<br />

B 1 N 1 N 2 (xyz) = B 1 N 1 (zyx) = B 1 (zxy) = yzx<br />

De <strong>la</strong> misma forma <strong>que</strong> antes, tenemos varias re<strong>la</strong>ciones, por ejemp<strong>lo</strong> B 1 N 2 = N 2 ya<br />

<strong>que</strong><br />

B 1 N 2 (xyz) = B 1 (zyx) = xzy = N 1 (xyz)<br />

Aná<strong>lo</strong>gamente se pueden deducir <strong>la</strong>s siguientes re<strong>la</strong>ciones (o reg<strong>la</strong>s <strong>para</strong> reemp<strong>la</strong>zar<br />

2 órdenes por 1 so<strong>la</strong>):<br />

II = I IB 1 = B 1 IB 2 = B 2 IN 1 = N 1 IN 2 = N 2 IN 3 = N 3<br />

B 1 I = B 1 B 1 B 1 = B 2 B 1 B 2 = I B 1 N 1 = N 3 B 1 N 2 = N 1 B 1 N 3 = N 2<br />

B 2 I = B 2 B 2 B 1 = I B 2 B 2 = B 1 B 2 N 1 = N 2 B 2 N 2 = N 3 B 2 N 3 = N 1<br />

N 1 I = N 1 N 1 B 1 = N 2 N 1 B 2 = N 3 N 1 N 1 = I N 1 N 2 = B 1 N 1 N 3 = B 2<br />

N 2 I = N 2 N 2 B 1 = N 3 N 2 B 2 = N 1 N 2 N 1 = B 2 N 2 N 2 = I N 2 N 3 = B 1<br />

N 3 I = N 3 N 3 B 1 = N 1 N 3 B 2 = N 2 N 3 N 1 = B 1 N 3 N 2 = B 2 N 3 N 3 = I<br />

No hay dificultad ahora en resolver el siguiente problema:<br />

Se tiene en el pizarrón una secuencia de letras. En cada turno, se pueden<br />

elegir 2 letras, borrar<strong>la</strong>s y escribir otra de acuerdo a <strong>la</strong>s siguientes reg<strong>la</strong>s<br />

AA → A AB → B AC → C AD → D AE → E AF → F<br />

BA → B BB → C BC → A BD → F BE → D BF → E<br />

CA → C CB → A CC → B CD → E CE → F CF → D<br />

DA → D DB → E DC → F DD → A DE → B DF → C<br />

EA → E EB → F EC → D ED → C EE → A EF → B<br />

F A → F F B → D F C → E F D → B F E → C F F → A<br />

Probar <strong>que</strong> sin importar <strong>la</strong>s elecciones <strong>que</strong> tomemos, <strong>la</strong> última letra <strong>que</strong><br />

<strong>que</strong>da es siempre <strong>la</strong> misma.<br />

34


Algunos problemas <strong>para</strong> pensar<br />

1. En el problema de <strong>la</strong>s cartas digamos <strong>que</strong> el peso de una carta b<strong>la</strong>nca es (−1) k<br />

donde k es <strong>la</strong> cantidad de cartas negras a su izquierda. Probar <strong>que</strong> en cada etapa,<br />

<strong>la</strong> suma de <strong>lo</strong>s pesos de <strong>la</strong>s cartas b<strong>la</strong>ncas aumenta o disminuye en una cantidad<br />

múltip<strong>lo</strong> de 3 o se mantiene igual. Usar esto <strong>para</strong> dar otra demostración.<br />

2. En un pizarrón hay cierta cantidad de letras e, a, b, c. En cada turno se pueden<br />

borrar 2 letras y escribir otra en función de <strong>la</strong> siguiente tab<strong>la</strong><br />

e a b c<br />

e e a b c<br />

a a e c b<br />

b b c e a<br />

c c b a e<br />

Demostrar <strong>que</strong> sin importar <strong>lo</strong> <strong>que</strong> hagamos, <strong>la</strong> última letra siempre es <strong>la</strong> misma.<br />

Sugerencia: Poner E(wxyz) = wxyz, A(wxyz) = xwzy, B(wxyz) = yzwx y<br />

C(wxyz) = zyxw (se puede pensar <strong>que</strong> si xyzw es un cuadrado entonces A y<br />

B es reflejar<strong>lo</strong> por sus bases medias y <strong>que</strong> C es reflejar<strong>lo</strong> por su centro).<br />

3. (XIV Olimpíada Riop<strong>la</strong>tense) Se desea co<strong>lo</strong>rear cada entero positivo con un<br />

co<strong>lo</strong>r utilizando <strong>la</strong> mayor cantidad posible de co<strong>lo</strong>res de manera <strong>que</strong> se verifi<strong>que</strong><br />

<strong>la</strong> siguiente condición: Si, en notación decimal, el número B se puede obtener<br />

a partir del número A, suprimiéndole a A dos dígitos iguales consecutivos (aa)<br />

o suprimiéndole a A cuatro dígitos consecutivos <strong>que</strong> formen dos pares iguales y<br />

consecutivas (abab), entonces A y B son del mismo co<strong>lo</strong>r.<br />

Por ejemp<strong>lo</strong>, 8, 833 y 22811 deben ser del mismo co<strong>lo</strong>r y también 72, 676772 y<br />

1173329898 son del mismo co<strong>lo</strong>r.<br />

Determinar cuál es <strong>la</strong> mayor cantidad de co<strong>lo</strong>res <strong>que</strong> se puede utilizar.<br />

35


2.8. Rompecabezas<br />

Ahora es el turno de un juego <strong>que</strong> ya era conocido en 1870. El rompecabezas,<br />

popu<strong>la</strong>rizado como “Fifteen Puzzle“, es de Noyes Chapman 10 .<br />

Se tiene un rompecabezas con forma de un tablero de 4 × 4 con 15 fichas<br />

numeradas 1, 2, ....., 15. Siempre debe haber una casil<strong>la</strong> vacía y el resto<br />

ocupadas por una ficha cada una. En cada turno podemos elegir una<br />

casil<strong>la</strong> vecina a <strong>la</strong> vacía y mover su ficha hasta ésta. ¿Es posible llegar de<br />

<strong>la</strong> posición de <strong>la</strong> izquierda a <strong>la</strong> de <strong>la</strong> derecha?<br />

1 2 3 4<br />

5 6 7 8<br />

9 10 11 12<br />

13 14 15<br />

1 2 3 4<br />

5 6 7 8<br />

9 10 11 12<br />

13 15 14<br />

Sea P 0 <strong>la</strong> posición inicial y P n a <strong>la</strong> <strong>que</strong> <strong>que</strong>remos llegar. Supongamos <strong>que</strong> existen<br />

posiciones intermedias P 1 , ..., P n−1 de forma <strong>que</strong> cada una se obtiene de <strong>la</strong> anterior<br />

empujando una ficha a <strong>la</strong> casil<strong>la</strong> vacía. Es fácil ver <strong>que</strong> n debe ser par. Para el<strong>lo</strong>,<br />

pintemos <strong>la</strong>s casil<strong>la</strong>s de b<strong>la</strong>nco y negro como <strong>la</strong>s de un tablero de ajedrez y notemos<br />

<strong>que</strong> <strong>la</strong> casil<strong>la</strong> vacía siempre cambia de co<strong>lo</strong>r. Como P 0 y P n tienen <strong>la</strong> misma casil<strong>la</strong><br />

vacía entonces al final del juego, ésta debe haber cambiado de co<strong>lo</strong>r una cantidad par<br />

de veces o <strong>lo</strong> <strong>que</strong> es <strong>lo</strong> mismo n debe ser par.<br />

a 1 a 2 a 3 a 4<br />

a 5 a 6 a 7 a 8<br />

a 9 a 10 a 11 a 12<br />

a 13 a 14 a 15 a 16<br />

Podemos pensar <strong>que</strong> <strong>la</strong> casil<strong>la</strong> vacía está ocupada por una ficha invisible con un<br />

16 y en cada turno podemos intercambiar esta ficha con cualquier vecina. Definamos<br />

<strong>para</strong> una posición P , con <strong>la</strong>s fichas como en <strong>la</strong> figura anterior, su signo como:<br />

sg P = sg ∏ i 0 y sg a = −1 si a < 0 (el producto nunca es cero<br />

ya <strong>que</strong> todos <strong>lo</strong>s a i son distintos entre sí). Vamos a usar el signo <strong>para</strong> llegar a un<br />

absurdo. Analicemos qué pasa con el signo después de cada turno.<br />

10 Aun<strong>que</strong> también algunos dicen <strong>que</strong> pertenece a Sam L<strong>lo</strong>yd<br />

36


Supongamos <strong>que</strong> P m+1 se obtiene de P m al intercambiar a i con a i+k (es decir <strong>que</strong><br />

a i = 16 o a i+k = 16, además por <strong>la</strong> forma del tablero k = 1 o k = 4). Para calcu<strong>la</strong>r<br />

sg P m+1 debemos calcu<strong>la</strong>r el signo de el producto de todas <strong>la</strong>s diferencias de <strong>lo</strong>s<br />

números en P m+1 . Este producto es muy parecido al <strong>que</strong> debemos calcu<strong>la</strong>r <strong>para</strong> P m ,<br />

<strong>la</strong> única diferencia es <strong>que</strong> <strong>lo</strong>s siguientes factores se dan vuelta:<br />

a i+1 − a i , a i+2 − a i , ...., a i+k−1 − a i , a i+k − a i , a i+k − a i+k−1 , ...., a i+k − a i+1<br />

Como 2k − 1 factores cambiaron por su opuesto, entonces<br />

De donde, recordando <strong>que</strong> n es par<br />

sg P m+1 = (−1) 2k−1 sg P m = −sg P m<br />

sg P n = −sg P n−1 = sg P n−2 = ... = −sg P 1 = sg P 0<br />

Es decir <strong>que</strong> sg P n = sg P 0 . Pero es fácil ver <strong>que</strong> sg P 0 = 1 y sg P n = −1, <strong>lo</strong> <strong>que</strong> es<br />

absurdo <strong>que</strong> viene de suponer <strong>que</strong> era posible llegar a <strong>la</strong> posición deseada. ‡<br />

Notemos <strong>que</strong> si bien el signo no es un invariante, éste varía de forma contro<strong>la</strong>da.<br />

Si <strong>que</strong>remos un invariante, <strong>para</strong> cada posición P pongamos I(P ) = sg P t(P ) donde<br />

t(P ) = 1 si <strong>la</strong> casil<strong>la</strong> vacía está en una casil<strong>la</strong> negra y t(P ) = −1 si está en una b<strong>la</strong>nca.<br />

Algunos problemas <strong>para</strong> pensar<br />

1. Supongamos <strong>que</strong> tenemos 2 posiciones <strong>para</strong> el rompecabezas del problema con<br />

el mismo signo y <strong>la</strong> casil<strong>la</strong> vacía en el mismo lugar. Probar <strong>que</strong> se puede llegar<br />

de una a <strong>la</strong> otra mediante <strong>la</strong>s operaciones permitidas.<br />

2. (Olimpíada Balkánica 2001) Se tiene un cubo de 3 × 3 × 3 dividido en 27 celdas<br />

unitarias. Una de <strong>la</strong>s celdas está vacía y el resto está ocupada por cubitos<br />

unitarios numerados 1, 2, ..., 26. Cada jugada consiste en empujar un cubito<br />

unitario a una celda vecina (<strong>que</strong> comparte una cara), si ésta se encuentra vacía.<br />

¿Es posible <strong>que</strong> después de algunas operaciones cada cubito numerado k esté<br />

en el lugar donde empezó el cubito 27 − k?<br />

3. El Cubo de Rubik es un cubo de 3 × 3 × 3 dividido en 27 cubitos unitarios con<br />

cada cara del cubo grande pintada de un co<strong>lo</strong>r distinto. En cada turno podemos<br />

rotar 90 o una cara a elección (es decir <strong>lo</strong>s 9 cubitos <strong>que</strong> <strong>la</strong> componen). ¿Es<br />

posible <strong>que</strong> después de algunos turnos volvamos a <strong>la</strong> posición original salvo por<br />

un único cubito unitario <strong>que</strong> intercambió dos de sus caras de lugar?<br />

37


4. (Lista Corta 40 IMO) Hay n chicos y cada uno tiene una pe<strong>lo</strong>ta con su nombre.<br />

En cierto momento <strong>lo</strong>s chicos comienzan a intercambiarse <strong>la</strong>s pe<strong>lo</strong>tas, es decir A<br />

se <strong>la</strong> da a B y B se <strong>la</strong> da a A. Se sabe <strong>que</strong> al final del día cada chico se fue con<br />

su pe<strong>lo</strong>ta y <strong>que</strong> cada par de chicos intercambiaron 1 vez <strong>la</strong> pe<strong>lo</strong>ta exactamente.<br />

Hal<strong>la</strong>r <strong>lo</strong>s posibles va<strong>lo</strong>res de n <strong>para</strong> <strong>lo</strong>s <strong>que</strong> esto es posible.<br />

5. Supongamos <strong>que</strong> tenemos n objetos, cada uno de el<strong>lo</strong>s con una eti<strong>que</strong>ta. En cada<br />

turno podemos intercambiar <strong>la</strong> eti<strong>que</strong>ta de cualesquiera 2 objetos. Demostrar<br />

<strong>que</strong> es imposible <strong>que</strong> después de una cantidad impar de turnos cada objeto<br />

tenga su eti<strong>que</strong>ta.<br />

6. Dada una permutación 11 π de 1, 2..., n definimos<br />

sg (π) = sg ∏ i


2.9. Pizarrones, Tab<strong>la</strong>s y Grupos<br />

Es momento de volver sobre algunos problemas<br />

En un pizarrón hay 10 signos + y 5 signos −. En cada turno se debe<br />

elegir dos de el<strong>lo</strong>s, borrar<strong>lo</strong>s y poner un + si eran iguales y un − si eran<br />

distintos. Probar <strong>que</strong> después de 14 turnos <strong>que</strong>dará un signo − so<strong>la</strong>mente.<br />

En un pizarrón hay cierta cantidad de letras a, b, c. En cada turno se puede<br />

reemp<strong>la</strong>zar una a y una b por una b, una a y una c por una c, una b y una<br />

c por una a, dos a por una a, dos b por una c o dos c por una b. El objetivo<br />

es dejar una so<strong>la</strong> letra. Probar <strong>que</strong> sin importar en qué orden realicemos<br />

<strong>la</strong>s operaciones, <strong>la</strong> letra <strong>que</strong> <strong>que</strong>de al final siempre será <strong>la</strong> misma.<br />

La información en <strong>lo</strong>s problemas anteriores se puede resumir en <strong>la</strong>s siguientes<br />

tab<strong>la</strong>s<br />

+ −<br />

+ + −<br />

− − +<br />

a b c<br />

a a b c<br />

b b c a<br />

c c a b<br />

Hagamos ahora el camino inverso y reconstruyamos el problema a partir de <strong>la</strong><br />

tab<strong>la</strong>.<br />

En un pizarrón hay cierta cantidad de letras e, a, b, c. En cada turno se<br />

pueden borrar 2 letras y escribir otra en función de <strong>la</strong> siguiente tab<strong>la</strong><br />

e a b c<br />

e e a b c<br />

a a e c b<br />

b b c e a<br />

c c b a e<br />

Demostrar <strong>que</strong> sin importar <strong>lo</strong> <strong>que</strong> hagamos, <strong>la</strong> última letra siempre es <strong>la</strong><br />

misma.<br />

39


Si ponemos un 1 por cada e y por cada a y un −1 por cada b y por cada c<br />

entonces estamos en <strong>la</strong>s mismas condiciones <strong>que</strong> en el primer problema, pues al borrar<br />

2 números <strong>lo</strong>s reemp<strong>la</strong>zamos por su producto. Por ejemp<strong>lo</strong><br />

a → 1<br />

b → −1<br />

c → −1<br />

Entonces podemos <strong>saber</strong> cuál será el último número, de donde podemos <strong>saber</strong> en<br />

qué par está <strong>la</strong> última letra entre {e, a} y {b, c}.<br />

De <strong>la</strong> misma forma, si ponemos un 1 por cada e y cada b y un −1 por cada a y<br />

cada c podemos <strong>saber</strong> en qué par estará <strong>la</strong> última letra entre {e, b} y {a, c}.<br />

Juntando <strong>la</strong> información anterior podemos <strong>saber</strong> cuál será <strong>la</strong> última letra; alcanza<br />

notar <strong>que</strong> el primer invariante nos dice en qué columna de <strong>la</strong> figura <strong>que</strong> sigue está y<br />

el segundo en qué fi<strong>la</strong>.<br />

e<br />

a<br />

b<br />

c<br />

Hemos probado entonces <strong>que</strong> sin importar <strong>lo</strong> <strong>que</strong> hagamos, <strong>la</strong> última letra siempre<br />

es <strong>la</strong> misma. ‡<br />

La solución anterior se puede reescribir en <strong>la</strong> siguiente forma más compacta. Reemp<strong>la</strong>cemos<br />

cada e por (1, 1), cada a por (1, −1), cada b por (−1, 1) y cada c por (−1, −1).<br />

Entonces en cada turno borramos 2 pares y <strong>lo</strong>s reemp<strong>la</strong>zamos por su producto (el producto<br />

de 2 pares es coordenada a coordenada). Por ejemp<strong>lo</strong><br />

a → ( 1, −1)<br />

b → (−1, 1)<br />

c → (−1, −1)<br />

Luego el producto de todos <strong>lo</strong>s pares es invariante y este resultado debe ser el par<br />

con el <strong>que</strong> se termina.<br />

De ahora en ade<strong>la</strong>nte por x ∗ y entendemos <strong>la</strong> letra de <strong>la</strong> tab<strong>la</strong> en <strong>la</strong> fi<strong>la</strong> de “x“ y<br />

en <strong>la</strong> columna de “y“ y <strong>lo</strong> l<strong>la</strong>mamos su ∗-producto. Por ejemp<strong>lo</strong>, a ∗ b = c, b ∗ b = e y<br />

e ∗ a = a.<br />

Por <strong>lo</strong> recién hecho, no só<strong>lo</strong> tiene sentido ∗-multiplicar 2 letras sino cualquier<br />

cantidad de letras. La ∗-multiplicación de un conjunto de letras es <strong>la</strong> última letra <strong>que</strong><br />

<strong>que</strong>da luego de aplicar el proceso del problema. Por ejemp<strong>lo</strong><br />

a ∗ b ∗ a ∗ c = (a ∗ b) ∗ (a ∗ c) = c ∗ b = e<br />

40


Lo anterior tiene sentido por<strong>que</strong> sin importar en qué orden pongamos <strong>lo</strong>s paréntesis<br />

y realicemos <strong>la</strong>s operaciones, siempre se llega al mismo resultado. En este contexto<br />

el ∗-producto de <strong>la</strong>s letras en el pizarrón es un invariante, pero no podíamos usar<strong>lo</strong><br />

<strong>para</strong> resolver el problema por<strong>que</strong> precisamos del problema <strong>para</strong> definir<strong>lo</strong>.<br />

El conjunto anterior con <strong>la</strong> operación ∗ es el grupo de Klein. En <strong>la</strong> sección <strong>que</strong><br />

viene vamos a utilizar<strong>lo</strong> <strong>para</strong> resolver algunos problemas pero es importante destacar<br />

<strong>que</strong> no hay nada en especial en éste, todo <strong>lo</strong> <strong>que</strong> podemos hacer con el grupo de Klein<br />

<strong>lo</strong> podemos hacer sin él.<br />

Cada tab<strong>la</strong> como <strong>la</strong>s anteriores define una operación entre ciertos elementos. Las<br />

tab<strong>la</strong>s <strong>que</strong> vimos en esta sección eran conmutativas, es decir <strong>que</strong> x ∗ y = y ∗ x <strong>para</strong><br />

cualquier par de elementos x, y. Sin embargo esto no es necesario como muestra <strong>la</strong><br />

siguiente tab<strong>la</strong> <strong>que</strong> resume el último problema de <strong>la</strong> sección anterior.<br />

∗ a b c d e f<br />

a a b c d e f<br />

b b c a f d e<br />

c c a b e f d<br />

d d e f a b c<br />

e e f d c a b<br />

f f d e b c a<br />

Cuando <strong>la</strong> tab<strong>la</strong> no es conmutativa ya no tiene sentido ∗-multiplicar conjuntos de<br />

letras sino secuencias (finitas) de letras. Veamos un ejemp<strong>lo</strong> usando <strong>la</strong> tab<strong>la</strong> anterior<br />

<strong>para</strong> ∗-multiplicar b ∗ d ∗ f ∗ e:<br />

b ∗ d ∗ f ∗ e = (b ∗ d) ∗ f ∗ e = f ∗ (f ∗ e) = f ∗ c = e<br />

b ∗ d ∗ f ∗ e = b ∗ (d ∗ f) ∗ e = (b ∗ c) ∗ e = a ∗ e = e<br />

b ∗ d ∗ f ∗ e = b ∗ d ∗ (f ∗ e) = b ∗ (d ∗ c) = b ∗ f = e<br />

Sin importar en qué orden hagamos <strong>la</strong>s operaciones siempre nos va a dar e, por<br />

eso es <strong>que</strong> decimos <strong>que</strong> b ∗ d ∗ f ∗ e = e. El hecho de <strong>que</strong> no importe el orden de<br />

<strong>la</strong>s operaciones es justamente el problema antes mencionado. De forma aná<strong>lo</strong>ga <strong>para</strong><br />

cada tab<strong>la</strong> podemos tratar de definir una operación. La pregunta es...... ¿Qué tab<strong>la</strong>s<br />

conducen a operaciones <strong>para</strong> <strong>la</strong>s <strong>que</strong> sin importar el orden en <strong>que</strong> <strong>la</strong>s realicemos<br />

siempre dan <strong>lo</strong> mismo?<br />

41


Algunos problemas <strong>para</strong> pensar<br />

1. Tenemos una secuencia de letras, en cada turno podemos elegir 2 letras consecutivas<br />

y cambiar<strong>la</strong>s por otra de acuerdo a <strong>la</strong> siguiente tab<strong>la</strong>:<br />

e a b c<br />

e e a b c<br />

a a b c d<br />

b b c e a<br />

c c d a e<br />

Demostrar <strong>que</strong> sin importar <strong>lo</strong> <strong>que</strong> hagamos <strong>la</strong> última letra siempre es <strong>la</strong> misma.<br />

2. Tenemos una secuencia de letras, en cada turno podemos elegir 2 letras consecutivas<br />

y cambiar<strong>la</strong>s por otra de acuerdo a una tab<strong>la</strong>. Sea x ∗ y el elemento en <strong>la</strong><br />

fi<strong>la</strong> de <strong>la</strong> “x“ y <strong>la</strong> columna de <strong>la</strong> “y“ y supongamos <strong>que</strong> <strong>para</strong> todo a, b, c:<br />

(a ∗ b) ∗ c = a ∗ (b ∗ c)<br />

Probar <strong>que</strong> sin importar <strong>lo</strong> <strong>que</strong> hagamos <strong>la</strong> última letra siempre es <strong>la</strong> misma.<br />

Nota: Esta propiedad caracteriza a <strong>la</strong>s “buenas tab<strong>la</strong>s“.<br />

3. En el pizarrón están escritos <strong>lo</strong>s números 1, 1, 1, .., 1 . En cada turno se deben<br />

2 3 n<br />

elegir 2 de el<strong>lo</strong>s, digamos a y b, borrar<strong>lo</strong>s y escribir en su lugar ab + a + b. ¿Qué<br />

número <strong>que</strong>da después de n − 1 turnos?<br />

4. (Rusia 1998) Una operación a ∗ b se define en el conjunto de <strong>lo</strong>s números reales<br />

tal <strong>que</strong> (a ∗ b) ∗ c = a + b + c probar <strong>que</strong> a ∗ b = a + b.<br />

5. (Leningrado 1989) Se tiene una operación a ∗ b entre <strong>lo</strong>s números enteros (es<br />

decir <strong>que</strong> a cada par de enteros a, b le asocia otro a ∗ b) tal <strong>que</strong> todo entero c es<br />

de <strong>la</strong> forma c = a ∗ b <strong>para</strong> algún par a, b de enteros. Probar <strong>que</strong> no es posible<br />

<strong>que</strong> (a ∗ b) ∗ c = a ∗ (b ∗ c) y a ∗ b = −b ∗ a.<br />

42


2.10. Senku<br />

El Senku es un juego del Sig<strong>lo</strong> XV III inventado por un aristócrata francés en su<br />

ais<strong>la</strong>miento en una celda de <strong>la</strong> Bastil<strong>la</strong>. Si bien se puede jugar en muchos tableros el<br />

objetivo es siempre el mismo: dejar <strong>la</strong> menor cantidad posible de fichas.<br />

Se tiene un tablero de 8 × 8 con una ficha en cada casil<strong>la</strong> salvo en <strong>la</strong>s<br />

esquinas. En cada turno debemos elegir 3 casil<strong>la</strong>s formando un rectángu<strong>lo</strong><br />

de 1 × 3 con un extremo vacío y <strong>la</strong>s otras 2 casil<strong>la</strong>s ocupadas. A continuación<br />

sacamos <strong>la</strong> ficha de <strong>la</strong> casil<strong>la</strong> del medio y a <strong>la</strong> ficha en un extremo<br />

<strong>la</strong> ponemos en el otro extremo.<br />

• • → •<br />

•<br />

• →<br />

•<br />

¿Es posible jugar de forma <strong>que</strong> <strong>que</strong>de una so<strong>la</strong> ficha en el tablero?<br />

Pongamos en cada casil<strong>la</strong> del tablero un elemento del grupo de Klein de <strong>la</strong> siguiente<br />

manera:<br />

a b c a b c a b<br />

c a b c a b c a<br />

b c a b c a b c<br />

a b c a b c a b<br />

c a b c a b c a<br />

b c a b c a b c<br />

a b c a b c a b<br />

c a b c a b c a<br />

No es difícil ver <strong>que</strong> el ∗-producto de <strong>lo</strong>s elementos del grupo de Klein en casil<strong>la</strong>s<br />

ocupadas por fichas se mantiene invariante. Inicialmente el producto es<br />

a 20 ∗ b 20 ∗ c 20 = e ∗ e ∗ e = e<br />

Si <strong>lo</strong>gramos dejar una so<strong>la</strong> ficha, ésta debe estar en una casil<strong>la</strong> con una e. Como<br />

ninguna casil<strong>la</strong> tiene una e, esto es imposible; entonces no es posible dejar só<strong>lo</strong> una<br />

ficha. ‡<br />

Como dijimos antes, no es necesario usar el grupo de Klein. Usando <strong>la</strong> misma<br />

distribución de letras <strong>que</strong> en <strong>la</strong> demostración anterior, notemos <strong>que</strong> en cada salto <strong>la</strong><br />

43


cantidad de letras a alterna su paridad, pues o borramos una a o agregamos una a.<br />

De <strong>la</strong> misma forma <strong>la</strong> cantidad de letras b alterna su paridad y <strong>la</strong> cantidad de letras<br />

c también <strong>lo</strong> hace. Como inicialmente hay 20 letras de cada una, entonces siempre<br />

deben tener <strong>la</strong> misma paridad, es decir o hay una cantidad par de cada letra o hay<br />

una cantidad impar de cada letra. Si <strong>lo</strong>gramos dejar una so<strong>la</strong> letra, entonces habrá<br />

una cantidad impar de una letra y una cantidad par de <strong>la</strong>s otras 2, <strong>lo</strong> <strong>que</strong> es absurdo.<br />

Otra alternativa es <strong>la</strong> siguiente: Consideremos por ejemp<strong>lo</strong> <strong>la</strong>s siguientes distribuciones<br />

de números<br />

1 1 1 1 1 1 1 −1<br />

−1 1 −1 −1 1 −1 −1 1<br />

−1 −1 1 −1 −1 1 −1 −1<br />

1 −1 −1 1 −1 −1 1 −1<br />

−1 1 −1 −1 1 −1 −1 1<br />

−1 −1 1 −1 −1 1 −1 −1<br />

1 −1 −1 1 −1 −1 1 −1<br />

−1 1 −1 −1 1 −1 −1 1<br />

−1 1 −1 −1 1 −1 −1 1<br />

−1 −1 1 −1 −1 1 −1 −1<br />

1 −1 −1 1 −1 −1 1 −1<br />

−1 1 −1 −1 1 −1 −1 1<br />

−1 −1 1 −1 −1 1 −1 −1<br />

1 −1 −1 1 −1 −1 1 −1<br />

−1 1 −1 −1 1 −1 −1 1<br />

−1 −1 1 −1 −1 1 −1 −1<br />

Como cada rectángu<strong>lo</strong> de 1 × 3 tiene dos −1 y un 1, en cada distribución el producto de<br />

<strong>lo</strong>s números en <strong>la</strong>s casil<strong>la</strong>s con fichas debe ser invariante. Inicialmente el producto es 1 en<br />

ambos, entonces de <strong>que</strong>dar una so<strong>la</strong> ficha ésta debe estar en una casil<strong>la</strong> con un 1 en ambos<br />

tableros. Pero como se puede ver, no hay ninguna casil<strong>la</strong> con un 1 en ambos tableros.<br />

La demostración anterior nos muestra como si bien a veces algunos invariantes por<br />

se<strong>para</strong>do no alcanzan, al juntar <strong>la</strong> información de ambos <strong>lo</strong>gramos resolver el problema. La<br />

demostración usando el grupo de Klein no es más <strong>que</strong> usar 2 invariantes al mismo tiempo.<br />

Algunos problemas <strong>para</strong> pensar<br />

1. Se tiene un tablero de 4 × 4 y 15 fichas. Se deben ubicar <strong>la</strong>s 15 fichas una en cada<br />

casil<strong>la</strong> dejando una casil<strong>la</strong> vacía y luego jugar al Senku. ¿Qué casil<strong>la</strong> nos conviene<br />

dejar vacía si <strong>que</strong>remos dejar só<strong>lo</strong> 1 ficha al final?<br />

2. (34 Imo 1993) Sobre un tablero de ajedrez infinito se juega de <strong>la</strong> siguiente manera: Al<br />

principio hay n 2 fichas dispuestas sobre el tablero en un cuadrado de n × n de casil<strong>la</strong>s<br />

adyacentes, con una ficha en cada casil<strong>la</strong>. Cada jugada es un salto de una ficha en<br />

dirección horizontal o vertical sobre una casil<strong>la</strong> adyacente, ocupada por otra, hasta<br />

una no ocupada, contigua a el<strong>la</strong>. La ficha sobre <strong>la</strong> <strong>que</strong> se ha saltado se retira (como<br />

en el Senku). Halle <strong>lo</strong>s va<strong>lo</strong>res de n <strong>para</strong> <strong>lo</strong>s <strong>que</strong> el juego puede terminar <strong>que</strong>dando<br />

una única ficha en el tablero.<br />

44


3. Se tiene un tablero como el de <strong>la</strong> figura con una ficha en cada casil<strong>la</strong> salvo en el centro.<br />

Las fichas se comen como en el Senku. ¿Es posible dejar 1 so<strong>la</strong> ficha y <strong>que</strong> ésta <strong>que</strong>de<br />

en <strong>la</strong> casil<strong>la</strong> marcada? ¿Y dejar una so<strong>la</strong> en el centro?<br />

4. Supongamos <strong>que</strong> tenemos un grafo p<strong>la</strong>no en donde todos <strong>lo</strong>s vértices tienen 3 aristas.<br />

Probar <strong>que</strong> es posible pintar <strong>la</strong>s aristas con 3 co<strong>lo</strong>res, de forma tal <strong>que</strong> no haya dos del<br />

mismo co<strong>lo</strong>r <strong>que</strong> tengan un vértice en común, si y só<strong>lo</strong> si se pueden pintar <strong>la</strong>s caras<br />

con 4 co<strong>lo</strong>res de forma <strong>que</strong> no haya 2 caras con un <strong>la</strong>do en común del mismo co<strong>lo</strong>r.<br />

45


2.11. Ranas<br />

El siguiente juego <strong>lo</strong> propuso J.H. Conway. Se mueve como en el Senku aun<strong>que</strong> también<br />

tiene un aire al problema “Piedras“.<br />

En un tablero infinito hay una línea <strong>que</strong> l<strong>la</strong>maremos horizonte. En algunas<br />

casil<strong>la</strong>s por debajo del horizonte hay ranas, a <strong>lo</strong> sumo una por casil<strong>la</strong>. Las<br />

ranas se mueven de <strong>la</strong> siguiente manera, se pueden mover en horizontal o en<br />

vertical saltando por encima de otra rana contigua siempre <strong>que</strong> <strong>la</strong> casil<strong>la</strong> a <strong>la</strong><br />

<strong>que</strong> salte esté vacía, comiendo <strong>la</strong> rana sobre <strong>la</strong> <strong>que</strong> saltó. ¿Es posible <strong>para</strong> alguna<br />

disposición inicial de <strong>la</strong>s ranas <strong>que</strong> una de el<strong>la</strong>s llegue a <strong>la</strong> quinta fi<strong>la</strong> por encima<br />

del horizonte?<br />

•<br />

Supongamos <strong>que</strong> tenemos una disposición inicial <strong>para</strong> <strong>la</strong>s ranas, totalmente contenida<br />

por debajo del horizonte, <strong>que</strong> les permita llegar a una casil<strong>la</strong> en <strong>la</strong> quinta fi<strong>la</strong> (a <strong>la</strong> <strong>que</strong><br />

l<strong>la</strong>maremos “casil<strong>la</strong> especial“). Podemos suponer <strong>que</strong> <strong>la</strong> cantidad de ranas es finita, pues <strong>la</strong>s<br />

involucradas <strong>para</strong> <strong>que</strong> una de el<strong>la</strong>s alcance <strong>la</strong> quinta fi<strong>la</strong> debe ser<strong>lo</strong>.<br />

√<br />

5−1<br />

2<br />

, <strong>que</strong> es <strong>la</strong> raíz positiva de <strong>la</strong> ecuación x 2 + x = 1. Notemos <strong>que</strong> 0 < w < 1<br />

Sea w =<br />

y escribamos en cada casil<strong>la</strong> una potencia de w de <strong>la</strong> siguiente forma: pongamos un 1 en <strong>la</strong><br />

casil<strong>la</strong> especial y en <strong>la</strong>s demás casil<strong>la</strong>s potencias de w como muestra <strong>la</strong> figura.<br />

1<br />

w 2 w w 2<br />

w 4 w 3 w 2 w 3 w 4<br />

w 6 w 5 w 4 w 3 w 4 w 5 w 6<br />

w 8 w 7 w 6 w 5 w 4 w 5 w 6 w 7 w 8<br />

w 9 w 8 w 7 w 6 w 5 w 6 w 7 w 8 w 9<br />

w 9 w 8 w 7 w 6 w 7 w 8 w 9<br />

w 9 w 8 w 7 w 8 w 9<br />

w 9 w 8 w 9<br />

46


El peso de una rana es el número en <strong>la</strong> casil<strong>la</strong> <strong>que</strong> ocupa y naturalmente el peso total<br />

es <strong>la</strong> suma de <strong>lo</strong>s pesos de todas <strong>la</strong>s ranas. La situación ideal sería <strong>que</strong> en cada salto el<br />

peso total se mantenga invariante. Si bien esto no es cierto, <strong>lo</strong> <strong>que</strong> podemos ver es <strong>que</strong> esta<br />

cantidad nunca crece.<br />

En efecto, supongamos <strong>que</strong> <strong>la</strong> rana sobre <strong>la</strong> <strong>que</strong> se salta tenía peso w n+1 y <strong>que</strong> no estaba<br />

en <strong>la</strong> misma columna <strong>que</strong> <strong>la</strong> casil<strong>la</strong> especial. Entonces <strong>la</strong> rana <strong>que</strong> saltó tenía peso w n+2 y<br />

pasó a tener peso w n o tenía peso w n y pasó a tener peso w n+2 . En ambos casos el peso total<br />

no crece, pues <strong>la</strong> rana sobre <strong>la</strong> <strong>que</strong> se saltó se retira y como w+w 2 = 1 → w n+1 +w n+2 = w n<br />

entonces:<br />

1. w n+1 + w n+2 ≥ w n<br />

2. w n + w n+1 ≥ w n+2<br />

Falta analizar qué pasa si se saltó sobre una rana en <strong>la</strong> columna de <strong>la</strong> casil<strong>la</strong> especial.<br />

Si el salto fue vertical, el mismo argumento <strong>que</strong> antes funciona; si el salto fue horizontal es<br />

más simple pues <strong>la</strong> rana <strong>que</strong> saltó seguirá teniendo el mismo peso pero <strong>la</strong> otra se retira.<br />

En conclusión, el peso total nunca crece.<br />

Ahora es momento de estimar el peso total inicial, <strong>para</strong> el<strong>lo</strong> debemos sumar todos <strong>lo</strong>s<br />

números debajo del horizonte. Vamos a usar 12 <strong>que</strong> w 3 + 2w 4 + 2w 5 + ... = 1 pues<br />

Luego<br />

w 3 + 2w 4 + 2w 5 + .... = w 3 + w 4 + w 4 + w 5 + w 5 + w 6 + ....<br />

w 3 + 2w 4 + 2w 5 + .... = w 2 + w 3 + w 4 + ... = w 2 1<br />

1 − w = 1<br />

1. Los números en <strong>la</strong> primer fi<strong>la</strong> suman w 5 + 2w 6 + 2w 7 + ... = w 2<br />

2. Los números en <strong>la</strong> segunda fi<strong>la</strong> suman w 6 + 2w 7 + 2w 8 + ... = w 3<br />

3. Los números en <strong>la</strong> tercer fi<strong>la</strong> suman w 7 + 2w 8 + 2w 9 + ... = w 4<br />

4. Los números en <strong>la</strong> cuarta fi<strong>la</strong> suman ........<br />

Entonces todos <strong>lo</strong>s números suman w 2 + w 3 + w 4 + .... = w 2 1<br />

1−w<br />

= 1. De donde, inicialmente<br />

el peso total de <strong>la</strong>s ranas era < 1 (<strong>para</strong> ser 1 necesitaríamos de infinitas ranas) <strong>lo</strong><br />

<strong>que</strong> es absurdo pues cuando una rana llegó a <strong>la</strong> casil<strong>la</strong> especial el<strong>la</strong> so<strong>la</strong> superó el peso total.<br />

Hemos probado entonces <strong>que</strong> no existe ninguna disposición inicial <strong>para</strong> <strong>la</strong>s ranas por debajo<br />

del horizonte <strong>que</strong> les permita llegar a <strong>la</strong> quinta fi<strong>la</strong>. ‡<br />

Algunos problemas <strong>para</strong> pensar<br />

1. Demostrar <strong>que</strong> el mínimo número de ranas <strong>para</strong> llegar a <strong>la</strong> tercer fi<strong>la</strong> sobre el horizonte<br />

es 8 y <strong>para</strong> llegar a <strong>la</strong> cuarta 20.<br />

12 Con respecto a <strong>la</strong>s “Sumas infinitas“ consultar el apéndice<br />

47


2.12. Monovariantes<br />

Siguiendo el ejemp<strong>lo</strong> de <strong>la</strong> sección anterior, veamos algunos casos en donde en vez de<br />

usar algo <strong>que</strong> se mantiene cuando modificamos <strong>la</strong> situación, usamos algo <strong>que</strong> siempre varía<br />

en forma monótona (es decir siempre crece o siempre decrece).<br />

En una fiesta hay 600 personas y 10 habitaciones, en cada segundo una persona<br />

se cambia de habitación de forma <strong>que</strong> pasa a una <strong>que</strong> tenía más personas <strong>que</strong><br />

en <strong>la</strong> <strong>que</strong> estaba. Probar <strong>que</strong> tarde o temprano todas <strong>la</strong>s personas estarán en <strong>la</strong><br />

misma habitación.<br />

Supongamos <strong>que</strong> una habitación tenía m personas y una de el<strong>la</strong>s se fue a otra <strong>que</strong> tenía<br />

n, con n ≥ m. Entonces (n + 1) 2 + (m − 1) 2 = n 2 + m 2 + 2(n − m + 1) ≥ n 2 + m 2 + 2,<br />

de donde <strong>la</strong> suma de <strong>lo</strong>s cuadrados de <strong>la</strong> cantidad de personas en cada habitación siempre<br />

crece estrictamente. Pero si l<strong>la</strong>mamos S a este número, S < 10 × 600 2 , entonces a partir de<br />

cierto momento, S deja de crecer. Entonces <strong>la</strong>s personas dejan de cambiar de habitación de<br />

donde todas están en <strong>la</strong> misma. ‡<br />

Dado un polígono no convexo le aplicamos <strong>la</strong> siguiente operación: elegimos 2<br />

vértices no consecutivos A y B, tales <strong>que</strong> el polígono está contenido en uno<br />

de <strong>lo</strong>s 2 semip<strong>la</strong>nos <strong>que</strong> determina <strong>la</strong> recta AB y se refleja una de <strong>la</strong>s partes<br />

del polígono <strong>que</strong> une A con B por el punto medio de AB. Si aplicamos esta<br />

operación indefinidamente, probar <strong>que</strong> tarde o temprano el polígono se vuelve<br />

convexo.<br />

En primer lugar, cada vez <strong>que</strong> aplicamos <strong>la</strong> operación al polígono no modificamos sus<br />

<strong>la</strong>dos ni sus direcciones, so<strong>la</strong>mente cambiamos el orden; luego <strong>lo</strong>s posibles polígonos <strong>que</strong><br />

podemos obtener son finitos.<br />

48


Por otro <strong>la</strong>do, el área del polígono siempre crece. Como <strong>lo</strong>s posibles polígonos son finitos<br />

entonces en algún momento tenemos <strong>que</strong> dejar de modificar<strong>lo</strong> pues como el área crece estos<br />

deben ser todos distintos. Pero si dejamos de modificar<strong>lo</strong> quiere decir <strong>que</strong> dejamos de aplicarle<br />

<strong>la</strong> operación o <strong>lo</strong> <strong>que</strong> es <strong>lo</strong> mismo <strong>que</strong> obtuvimos un polígono convexo. ‡<br />

Si en vez de reflejar por el punto medio de AB <strong>lo</strong> hacemos por <strong>la</strong> recta AB entonces<br />

el problema sigue valiendo pero es más difícil. Se conjetura <strong>que</strong> si el polígono inicial tiene<br />

n <strong>la</strong>dos entonces 2n reflexiones como <strong>la</strong>s anteriores alcanzan <strong>para</strong> transformar<strong>lo</strong> en uno<br />

convexo.<br />

Supongamos <strong>que</strong> tenemos una secuencia de números enteros no todos iguales<br />

S = (a, b, c, d). En cada turno <strong>lo</strong>s reemp<strong>la</strong>zamos por (a − b, b − c, c − d, d − a).<br />

Probar <strong>que</strong> <strong>para</strong> todo N, el módu<strong>lo</strong> de alguna coordenada se volverá más grande<br />

<strong>que</strong> N.<br />

Sea S n = (a n , b n , c n , d n ) <strong>la</strong> secuencia <strong>que</strong> se obtiene luego de n turnos. Usando <strong>que</strong><br />

a n + b n + c n + d n = 0 si n ≥ 1 se puede ver <strong>que</strong><br />

a 2 n+1 + b 2 n+1 + c 2 n+1 + d 2 n+1 = 2(a 2 n + b 2 n + c 2 n + d 2 n) + (a n + c n ) 2 + (b n + d n ) 2<br />

Entonces a 2 n+1 + b2 n+1 + c2 n+1 + d2 n+1 ≥ 2(a2 n + b 2 n + c 2 n + d 2 n) de donde<br />

a 2 n+1 + b 2 n+1 + c 2 n+1 + d 2 n+1 ≥ 2 n (a 2 1 + b 2 1 + c 2 1 + d 2 1)<br />

Ahora si tomamos n tal <strong>que</strong> 2 n (a 2 1 + b2 1 + c2 1 + d2 1 ) > 4N 2 entonces al menos uno entre<br />

(a n+1 , b n+1 , c n+1 , d n+1 ) debe tener módu<strong>lo</strong> más grande <strong>que</strong> N. ‡<br />

Supongamos <strong>que</strong> tenemos una secuencia de números enteros S = (a, b, c, d). En<br />

cada turno <strong>lo</strong>s reemp<strong>la</strong>zamos por (|a − b|, |b − c|, |c − d|, |d − a|). Probar <strong>que</strong><br />

tarde o temprano llegamos a (0, 0, 0, 0).<br />

Sea S n = (a n , b n , c n , d n ) <strong>la</strong> secuencia <strong>que</strong> se obtiene luego de n turnos y pongamos<br />

M i = max(a i , b i , c i , d i ). Notemos <strong>que</strong> M i+1 ≤ M i con igualdad si y só<strong>lo</strong> si el mayor de todos<br />

tiene un vecino <strong>que</strong> es 0. No es difícil ver <strong>que</strong> si M i > 0 entonces M i+4 < M i . Como M n ≥ 0<br />

<strong>para</strong> todo n entonces <strong>la</strong> secuencia {M n } debe decrecer hasta llegar a 0. Pero si M n = 0<br />

entonces (a n , b n , c n , d n ) = (0, 0, 0, 0) como <strong>que</strong>ríamos. ‡<br />

49


Se tiene un tablero infinito con n casil<strong>la</strong>s pintadas de negro. En cada turno se<br />

puede elegir una casil<strong>la</strong> y si tiene al menos 2 de sus 4 vecinas del co<strong>lo</strong>r opuesto<br />

le cambiamos el co<strong>lo</strong>r. Supongamos <strong>que</strong> luego de varios turnos hay 2006 casil<strong>la</strong>s<br />

pintadas de negro de forma <strong>que</strong> ningún par de el<strong>la</strong>s son vecinas y el resto están<br />

de b<strong>la</strong>nco. Probar <strong>que</strong> n ≥ 2006.<br />

Sea P el perímetro de <strong>la</strong>s casil<strong>la</strong>s negras, esto es <strong>la</strong> cantidad de <strong>la</strong>dos de casil<strong>la</strong>s negras<br />

<strong>que</strong> comparten con alguna casil<strong>la</strong> b<strong>la</strong>nca (dicho de otra forma, P es <strong>la</strong> cantidad de pares de<br />

casil<strong>la</strong>s vecinas de distinto co<strong>lo</strong>r). En cada turno P se mantiene igual o se hace más chico.<br />

Pues por ejemp<strong>lo</strong>, si pintamos una casil<strong>la</strong> b<strong>la</strong>nca de negro le estamos quitando al menos 2<br />

<strong>la</strong>dos al perímetro de <strong>la</strong>s negras y a <strong>lo</strong> sumo le estamos agregando 2 nuevos (recordar <strong>que</strong><br />

una casil<strong>la</strong> b<strong>la</strong>nca se puede pintar de negro só<strong>lo</strong> si tenía al menos 2 vecinas negras).<br />

Inicialmente P es a <strong>lo</strong> sumo 4n. En <strong>la</strong> situación final el perímetro es 4 × 2006. Luego<br />

4n ≥ 4 × 2006 es decir <strong>que</strong> n ≥ 2006. ‡<br />

Algunos problemas <strong>para</strong> pensar<br />

1. (San Petersburgo 1996) Hay varios números enteros escritos en el pizarrón, en cada<br />

turno se deben elegir 2 de el<strong>lo</strong>s, digamos a y b y se <strong>lo</strong>s reemp<strong>la</strong>za por (a, b) y [a, b].<br />

Demostrar <strong>que</strong> luego de cierta cantidad de turnos <strong>lo</strong>s números dejarán de cambiar.<br />

2. (MOP 1998) Se tiene un 2000 − agono regu<strong>la</strong>r y 2001 semil<strong>la</strong>s distribuidas entre sus<br />

vértices. En cada turno se debe elegir un vértice con al menos 2 semil<strong>la</strong>s, quitar<strong>la</strong>s y<br />

poner una de el<strong>la</strong>s en un vértice vecino y <strong>la</strong> otra en el otro vértice vecino. Probar <strong>que</strong><br />

tarde o temprano habrá al menos 1001 vértices sin semil<strong>la</strong>s.<br />

3. Probar <strong>que</strong> <strong>lo</strong>s vértices de cualquier grafo se pueden pintar de b<strong>la</strong>nco y negro de forma<br />

<strong>que</strong> <strong>para</strong> cada vértice, al menos <strong>la</strong> mitad de sus vecinos tiene el co<strong>lo</strong>r opuesto.<br />

4. (Irán TST 2005) Se tiene una cantidad finita de puntos en el p<strong>la</strong>no y un círcu<strong>lo</strong>. En<br />

cada turno se calcu<strong>la</strong> el baricentro de <strong>lo</strong>s puntos en el interior del círcu<strong>lo</strong> y se mueve<br />

su centro a este punto. Probar <strong>que</strong> luego de varios turnos el círcu<strong>lo</strong> va a dejar de<br />

moverse.<br />

5. (Rumania TST 2002) <strong>Todo</strong> miembro del par<strong>la</strong>mento tiene su rating personal. El par<strong>la</strong>mento<br />

se divide en grupos y cada miembro tiene un rating re<strong>la</strong>tivo, <strong>que</strong> es el cociente<br />

de su rating personal sobre <strong>la</strong> suma de <strong>lo</strong>s ratings personales de todos <strong>lo</strong>s miembros<br />

del grupo. Un miembro del par<strong>la</strong>mento se puede cambiar de un grupo a otro só<strong>lo</strong> si<br />

en el nuevo grupo su rating re<strong>la</strong>tivo aumenta. Cada día, so<strong>la</strong>mente un miembro del<br />

par<strong>la</strong>mento puede cambiar de grupo. Demostrar <strong>que</strong> só<strong>lo</strong> son posibles un número finito<br />

de movimientos.<br />

6. Se tiene un tablero de 10 × 10 con algunas casil<strong>la</strong>s infectadas. En cada segundo cada<br />

casil<strong>la</strong> con al menos 2 vecinas infectadas se infecta (2 casil<strong>la</strong>s son vecinas si tienen un<br />

<strong>la</strong>do en común). Si inicialmente había 9 casil<strong>la</strong>s infectadas, ¿es posible <strong>que</strong> <strong>la</strong> infección<br />

llegue a todo el tablero?<br />

50


2.13. Un problema y 2 soluciones<br />

El tercer problema de <strong>la</strong> 27 Imo de 1986 decía así:<br />

En cada vértice de un pentágono hay escrito un número entero de forma <strong>que</strong><br />

<strong>la</strong> suma de todos es positiva. Si x, y, z están en vértices consecutivos con y < 0<br />

entonces <strong>lo</strong>s podemos cambiar por x + y, −y, y + z. La operación anterior se<br />

repite siempre <strong>que</strong> se pueda. Probar <strong>que</strong> en algún momento todos <strong>lo</strong>s números<br />

se van a volver positivos.<br />

Sea s <strong>la</strong> suma de <strong>lo</strong>s 5 números escritos, s > 0 y como x + y + z = x + y − y + y + z<br />

entonces s se mantiene invariante con cada operación.<br />

Para cada pentágono con <strong>lo</strong>s números x 1 , x 2 , x 3 , x 4 , x 5 en sus vértices vamos a decir <strong>que</strong><br />

su “va<strong>lo</strong>r“ es<br />

(x 1 − x 3 ) 2 + (x 3 − x 5 ) 2 + (x 5 − x 2 ) 2 + (x 2 − x 4 ) 2 + (x 4 − x 1 ) 2<br />

Si v n es el va<strong>lo</strong>r del pentágono después de n operaciones entonces v n+1 < v n . Pues si por<br />

ejemp<strong>lo</strong> el cambio fue (x 2 , x 3 , x 4 ) → (x 2 + x 3 , −x 3 , x 3 + x 4 ) (<strong>lo</strong>s demás casos son aná<strong>lo</strong>gos)<br />

entonces<br />

v n = (x 1 − x 3 ) 2 + (x 3 − x 5 ) 2 + (x 5 − x 2 ) 2 + (x 2 − x 4 ) 2 + (x 4 − x 1 ) 2<br />

v n+1 = (x 1 −(−x 3 )) 2 +(−x 3 −x 5 ) 2 +(x 5 −(x 2 +x 3 )) 2 +((x 2 +x 3 )−(x 4 +x 3 )) 2 +((x 3 +x 4 )−x 1 ) 2<br />

Luego v n+1 − v n = 2sx 3 < 0 (cuestión de hacer <strong>la</strong> cuenta). Ahora es fácil terminar el<br />

problema, alcanza notar <strong>que</strong> <strong>la</strong> sucesión de enteros no negativos {v n } decrece indefinidamente,<br />

<strong>lo</strong> <strong>que</strong> es imposible. Entonces debe ser finita, es decir <strong>que</strong> só<strong>lo</strong> se pueden realizar una<br />

cantidad finita de operaciones o <strong>lo</strong> <strong>que</strong> es <strong>lo</strong> mismo <strong>que</strong> después de una cantidad finita de<br />

pasos el pentágono tendrá todos <strong>lo</strong>s vértices con números positivos. ‡<br />

La solución anterior, si bien interesante y muy ingeniosa, no nos dice nada acerca de<br />

qué ocurre en el caso en <strong>que</strong> <strong>lo</strong>s números no sean necesariamente enteros y nos da poca<br />

información acerca de cuántas operaciones hay <strong>que</strong> realizar o cuál es <strong>la</strong> posición final. Veamos<br />

otra solución <strong>que</strong> se acerca más a estas preguntas.<br />

Pongamos x 1 = x 6 = x 11 = ... , x 2 = x 7 = ... , ...., x 5 = x 10 = ... y consideremos el<br />

conjunto infinito T de sumas de <strong>la</strong> forma x i + x i+1 + ... + x j con 1 ≤ i ≤ 5 y i ≤ j, por<br />

ejemp<strong>lo</strong><br />

x 3 + x 4 + x 5 + x 6 + x 7 + x 8 + x 9 + x 10 + x 11<br />

En cada cambio del tipo (x 2 , x 3 , x 4 ) → (x 2 + x 3 , −x 3 , x 3 + x 4 ) el conjunto T se mantiene<br />

igual salvo <strong>que</strong> x 3 cambia por −x 3 . Lo <strong>que</strong> pasa es <strong>que</strong> cada suma se mantiene o se intercambia<br />

con otra, por ejemp<strong>lo</strong><br />

x 3 + x 4 + x 5 → (−x 3 ) + (x 3 + x 4 ) + x 5 = x 4 + x 5<br />

51


x 4 + x 5 → (x 3 + x 4 ) + x 5 = x 3 + x 4 + x 5<br />

Es decir <strong>que</strong> en cada paso un elemento negativo del conjunto T se cambia a positivo<br />

y el resto <strong>que</strong>da igual. Si bien el conjunto es infinito, como s > 0, so<strong>la</strong>mente puede tener<br />

una cantidad finita de elementos negativos. Entonces a <strong>lo</strong> sumo podemos realizar finitas<br />

operaciones, es decir <strong>que</strong> después de una cantidad finita de cambios todos <strong>lo</strong>s números serán<br />

positivos.<br />

Algunos problemas <strong>para</strong> pensar<br />

1. Probar <strong>que</strong> en el problema anterior, no importa qué operaciones hagamos, siempre se<br />

tarda <strong>la</strong> misma cantidad de pasos en <strong>lo</strong>grar <strong>que</strong> todos sean positivos.<br />

2. (Olimpíada de Leningrado 1990) Son dados 20 números en un círcu<strong>lo</strong>. Si x, y, z son<br />

números consecutivos, se <strong>lo</strong>s puede cambiar por x + y, −y, y + z. ¿Es posible cambiar<br />

<strong>la</strong> 20−up<strong>la</strong> (1, 2, 3, 4, 5, 6, 7, 8, 9, 10, −1, −2, −3, −4, −5, −6, −7, −8, −9, −10) por<br />

(10, 9, 8, 7, 6, 5, 4, 3, 2, 1, −10, −9, −8, −7, −6, −5, −4, −3, −2, −1) usando estas operaciones?<br />

3. (Olimpíada de Leningrado 1990) Son dados varios números en un círcu<strong>lo</strong>. Si x, y, z son<br />

números consecutivos, se <strong>lo</strong>s puede cambiar por x + y, −y, y + z. Probar <strong>que</strong> luego de<br />

una cantidad finita de estas operaciones se puede conseguir <strong>que</strong> todos sean positivos<br />

y demostrar <strong>que</strong> <strong>la</strong> posición final es única.<br />

52


2.14. Diamante Azteca<br />

Supongamos <strong>que</strong> tenemos un tablero de 2n × 2n cuyas fi<strong>la</strong>s están numeradas de abajo<br />

hacia arriba −n, .., −1, 1, ..., n y sus columnas de izquierda a derecha −n, ..., −1, 1, .., n (notar<br />

<strong>que</strong> no hay ni fi<strong>la</strong> ni columna 0). La casil<strong>la</strong> (x, y) es <strong>la</strong> <strong>que</strong> está en <strong>la</strong> columna x y <strong>la</strong> fi<strong>la</strong><br />

y. El Diamante Azteca de orden n es <strong>la</strong> figura formada por <strong>la</strong>s casil<strong>la</strong>s (x, y) tales <strong>que</strong><br />

|x| + |y| ≤ n + 1.<br />

Cada vez <strong>que</strong> digamos <strong>que</strong> el Diamante está cubierto por fichas de dominó (cada una de<br />

<strong>la</strong>s cuales cubre 2 casil<strong>la</strong>s <strong>que</strong> comparten un <strong>la</strong>do) estamos asumiendo implícitamente <strong>que</strong><br />

<strong>la</strong>s fichas no se superponen ni salen fuera del Diamante.<br />

Figura 1: Diamante Azteca de orden 4 y rotaciones permitidas<br />

Se tiene un Diamante Azteca de orden n cubierto por fichas de dominó verticales.<br />

En cada turno podemos rotar 90 o cualquier cuadrado de 2 × 2 cubierto por<br />

exactamente 2 dominós. Probar <strong>que</strong> <strong>la</strong> menor cantidad de rotaciones necesarias<br />

<strong>para</strong> <strong>lo</strong>grar <strong>que</strong> todos <strong>lo</strong>s dominós <strong>que</strong>den horizontales es n(n+1)(2n+1)<br />

6<br />

.<br />

En primer lugar veamos como ir de “todos verticales“ a “todos horizontales“ y tratemos<br />

de contar cuántas rotaciones usamos. Nos será de gran ayuda <strong>la</strong> siguiente observación:<br />

“Supongamos <strong>que</strong> tenemos 2 dominós verticales a <strong>la</strong> misma altura se<strong>para</strong>dos por 2k − 2<br />

columnas. Si el espacio entre el<strong>lo</strong>s está completamente cubierto por dominós horizontales<br />

entonces con 2k − 1 rotaciones podemos <strong>lo</strong>grar <strong>que</strong> <strong>lo</strong>s 2k dominós estén horizontales.“<br />

2k Columnas<br />

53


Para ir de “todos verticales“ a “todos horizontales“ <strong>la</strong> estrategia es <strong>la</strong> siguiente: en <strong>la</strong> primer<br />

etapa nos ocupamos de poner <strong>lo</strong>s dominós en <strong>la</strong>s columnas −1 y 1 horizontales, en <strong>la</strong> segunda<br />

tenemos <strong>que</strong> <strong>lo</strong>grar <strong>que</strong> <strong>la</strong>s columnas −2, −1, 1, 2 <strong>que</strong>den cubiertas por dominós horizontales<br />

y el resto por dominós verticales, .... , en <strong>la</strong> k−ésima etapa tenemos <strong>que</strong> <strong>lo</strong>grar <strong>que</strong> <strong>la</strong>s<br />

columnas −k, ...., k <strong>que</strong>den cubiertas por dominós horizontales y el resto por verticales.<br />

Después de <strong>la</strong> n−ésima etapa todas <strong>la</strong>s columnas estarán cubiertas por dominós horizontales<br />

y habremos conseguido llegar a “todos horizontales“. Veamos <strong>que</strong> esto es posible y calculemos<br />

cuántas rotaciones usamos.<br />

Figura 2: Etapa 1 y par <strong>para</strong> <strong>la</strong> Etapa 2<br />

Es fácil ver <strong>que</strong> en <strong>la</strong> primer etapa usamos so<strong>la</strong>mente n rotaciones. En <strong>la</strong> segunda etapa,<br />

asociemos a cada dominó vertical en <strong>la</strong> columna −2 otro en <strong>la</strong> columna 2 a <strong>la</strong> misma<br />

altura (por ejemp<strong>lo</strong>, <strong>lo</strong>s dominós marcados en <strong>la</strong> figura 2). Como el espacio entre el<strong>lo</strong>s<br />

está completamente cubierto por dominós horizontales entonces, por <strong>la</strong> observación, en 3<br />

rotaciones podemos hacer <strong>que</strong> estén todos horizontales. Como hay n − 1 pares, uno por cada<br />

dominó vertical en <strong>la</strong> columna −2, entonces en esta etapa usamos 3(n − 1) rotaciones.<br />

Figura 3: Etapa 3 y pares de dominós a <strong>la</strong> misma altura asociados<br />

54


Al principio de <strong>la</strong> k−ésima etapa tenemos <strong>la</strong>s columnas −(k − 1), ...., k − 1 cubiertas por<br />

dominós horizontales y el resto por dominós verticales. Hay n − k + 1 dominós verticales en<br />

<strong>la</strong> columna −k, a cada uno de el<strong>lo</strong>s <strong>lo</strong> asociamos a otro dominó vertical a <strong>la</strong> misma altura en<br />

<strong>la</strong> columna k (como en <strong>la</strong> figura 3). Como entre el<strong>lo</strong>s hay 2k − 2 columnas completamente<br />

cubiertas por dominós horizontales entonces en 2k − 1 rotaciones podemos hacer <strong>que</strong> estén<br />

todos horizontales. Como hay n − k + 1 pares precisamos (2k − 1)(n − k + 1) rotaciones.<br />

Notemos <strong>que</strong> al finalizar esta etapa, <strong>la</strong>s columnas −k, ..., k están completamente cubiertas<br />

por dominós horizontales y el resto por dominós verticales.<br />

Para <strong>saber</strong> cuantas rotaciones usamos en total debemos sumar (2k −1)(n−k +1) <strong>para</strong> k<br />

entre 1 y n ya <strong>que</strong> después de n etapas habremos llegado a “todos horizontales“. La cantidad<br />

de rotaciones usadas es 13 n∑<br />

n(n + 1)(2n + 1)<br />

(2k − 1)(n − k + 1) =<br />

6<br />

k=1<br />

Veamos <strong>que</strong> en efecto <strong>la</strong> cantidad anterior es <strong>la</strong> menor posible. Para el<strong>lo</strong> escribamos en<br />

cada casil<strong>la</strong> un número como muestra <strong>la</strong> figura. El rango de un cubrimiento del Diamante<br />

Azteca por dominós es <strong>la</strong> suma de <strong>lo</strong>s números en <strong>la</strong>s casil<strong>la</strong>s inferior izquierda de cada<br />

dominó. ¿Qué pasa con el rango después de cada rotación?<br />

Figura 4: El cubrimiento de <strong>la</strong> derecha tiene rango 16<br />

En cada rotación, el rango crece o disminuye en 1. Para probar <strong>lo</strong> anterior alcanza con<br />

analizar <strong>la</strong> figura 5. Supongamos <strong>que</strong> <strong>lo</strong>s dominós <strong>que</strong> rotamos tenían en su casil<strong>la</strong> inferior<br />

izquierda uno el número a y el otro b y pasaron a tener uno el número b y el otro c. Como <strong>la</strong>s<br />

casil<strong>la</strong>s en una misma diagonal <strong>que</strong> comparten un vértice tienen números consecutivos, es<br />

decir |a − c| = 1, entonces el rango creció en 1 o disminuyó en 1 (dependiendo de si a = c + 1<br />

o a = c − 1).<br />

13 Consultar el apéndice “Sumas de Potencias“<br />

55


Figura 5: Variación del rango en una rotación<br />

Es fácil ver <strong>que</strong> el rango de “todos horizontales“ es 0 pues cada ficha aporta <strong>lo</strong> opuesto<br />

<strong>que</strong> <strong>la</strong> ficha simétrica con respecto al eje de simetría horizontal del Diamante. Por otro <strong>la</strong>do,<br />

calcu<strong>la</strong>r el rango de “todos verticales“ es ligeramente mas complicado. En <strong>la</strong> mitad izquierda<br />

del Diamante <strong>la</strong> columna −k aporta −(n−k +1) 2 al rango y en <strong>la</strong> mitad derecha <strong>la</strong> columna<br />

k aporta (n − k + 1)(n + k). Luego el rango de “todos verticales“ es 14 :<br />

n∑<br />

(n − k + 1)(n + k) −<br />

k=1<br />

n∑<br />

(n − k + 1) 2 =<br />

k=1<br />

n∑<br />

(n − k + 1)(n + k) −<br />

k=1<br />

n∑<br />

(n − k + 1)(2k − 1)<br />

k=1<br />

n∑<br />

(n − k + 1) 2 =<br />

k=1<br />

n(n + 1)(2n + 1)<br />

6<br />

Como con cada rotación el rango a <strong>lo</strong> sumo disminuye en 1, <strong>para</strong> ir de “todos verticales“<br />

a “todos horizontales“ precisamos al menos n(n+1)(2n+1)<br />

6<br />

rotaciones. ‡<br />

No es difícil probar <strong>que</strong> <strong>para</strong> todo cubrimiento T de rango r(T ) valen <strong>la</strong>s desigualdades:<br />

n(n + 1)(2n + 1)<br />

0 ≤ r(T ) ≤<br />

6<br />

Más aun, el único cubrimiento de rango nu<strong>lo</strong> es “todos horizontales“ y el único cubrimiento<br />

de rango máximo es “todos verticales“. Por otro <strong>la</strong>do, r(T ) no apareció en <strong>la</strong> solución<br />

anterior por casualidad. Se puede probar <strong>que</strong> <strong>la</strong> menor cantidad de rotaciones <strong>para</strong> ir de<br />

un cubrimiento T hasta “todos horizontales“ es r(T ), su rango. Esto es remarcable, pues en<br />

principio ni siquiera sabemos <strong>que</strong> sea cierto <strong>que</strong> se puede llegar en un número arbitrario de<br />

rotaciones.<br />

Notemos <strong>que</strong> <strong>la</strong> noción de “rango“ de un cubrimiento no só<strong>lo</strong> nos dice cuál es <strong>la</strong> menor<br />

cantidad de rotaciones necesarias <strong>para</strong> <strong>que</strong> todos <strong>que</strong>den horizontales sino <strong>que</strong> nos proporciona<br />

de una forma <strong>para</strong> hacer<strong>lo</strong>, alcanza con siempre elegir <strong>para</strong> rotar un cuadrado <strong>que</strong><br />

haga bajar el rango en 1.<br />

Otro problema re<strong>la</strong>cionado al Diamante Azteca es el de determinar, <strong>para</strong> cada n, <strong>la</strong><br />

cantidad de cubrimientos por fichas de dominó del Diamante Azteca de orden n. Podemos<br />

14 Consultar el apéndice antes mencionado<br />

56


ir más lejos todavía y preguntarnos: ¿Será posible calcu<strong>la</strong>r, <strong>para</strong> cada r y cada n, cuántos<br />

cubrimientos del Diamante Azteca de orden n hay de rango r?<br />

Antes de dar una respuesta a <strong>la</strong> pregunta anterior es preciso hacer una pe<strong>que</strong>ña observación.<br />

En “todos horizontales“ no hay ningún dominó vertical y en cada rotación o <strong>para</strong>mos<br />

2 dominó o acostamos 2 dominó, luego <strong>la</strong> cantidad de dominós verticales debe ser siempre<br />

par. Como desde “todos horizontales“ podemos llegar a cualquier cubrimiento por rotaciones<br />

entonces <strong>para</strong> cualquier cubrimiento <strong>la</strong> cantidad de dominós verticales es par.<br />

Dado un cubrimiento T , por r(T ) denotamos su rango y por v(T ) <strong>la</strong> mitad de <strong>la</strong> cantidad<br />

de dominós verticales en T . En <strong>la</strong> siguiente suma, T recorre todos <strong>lo</strong>s cubrimientos del<br />

Diamante Azteca de orden n por dominós.<br />

P n (x, y) = ∑ T<br />

x v(T ) y r(T )<br />

El polinomio P n (x, y) no es más <strong>que</strong> una forma de guardar información. El coeficiente<br />

de x v y r de P n (x, y) es <strong>la</strong> cantidad de cubrimientos con 2v dominós verticales y rango r. La<br />

razón <strong>para</strong> guardar esta información de esta extraña manera es <strong>que</strong> P n (x, y) es más fácil de<br />

recordar <strong>que</strong> cada uno de sus coeficientes pues 15 :<br />

n−1<br />

∏<br />

P n (x, y) = (1 + xy 2k+1 ) n−k<br />

k=0<br />

Veamos como usar <strong>la</strong> formu<strong>la</strong> anterior. Si por ejemp<strong>lo</strong> deseamos <strong>saber</strong> cuántos cubrimientos<br />

del Diamante Azteca de orden n hay, debemos poner x = y = 1 pues de esa forma<br />

cada cubrimiento contribuye con 1 en <strong>la</strong> formu<strong>la</strong> de P n (x, y), luego<br />

n−1<br />

∏<br />

P n (1, 1) = (1 + 1 2k+1 ) n−k =<br />

k=0<br />

n−1<br />

∏<br />

k=0<br />

2 n−k = 2 n(n+1)<br />

2<br />

Es decir <strong>que</strong> hay 2 n(n+1)<br />

2 cubrimientos. Si ponemos x = 1, entonces cada cubrimiento<br />

de rango r contribuye con y r a P n (1, y), luego <strong>la</strong> cantidad de cubrimientos de rango r es el<br />

coeficiente de y r de P n (1, y).<br />

n−1<br />

∏<br />

P n (1, y) = (1 + y 2k+1 ) n−k<br />

k=0<br />

15 En realidad es más sutil; si por ejemp<strong>lo</strong> <strong>que</strong>remos <strong>saber</strong> cuántos cubrimientos hay, alcanza con<br />

evaluar el polinomio en un va<strong>lo</strong>r dado, pero si só<strong>lo</strong> tuviéramos <strong>la</strong> lista de sus coeficientes, sumar<strong>lo</strong>s<br />

podría resultar demasiado complicado. Por otro <strong>la</strong>do, a veces es más fácil obtener <strong>la</strong> expresión del<br />

polinomio <strong>que</strong> cada uno de sus coeficientes por se<strong>para</strong>do. El último problema es un ejemp<strong>lo</strong> de esta<br />

situación.<br />

57


Por último, si ponemos y = 1 entonces cada cubrimiento con 2v dominós verticales<br />

contribuye con x v a P n (x, 1).<br />

n−1<br />

∏<br />

P n (x, 1) = (1 + x) n−k = (1 + x) n(n+1)<br />

2<br />

k=0<br />

Algunos problemas <strong>para</strong> pensar<br />

1. (San Petersburgo 1997) Se tiene el Diamante Azteca de orden n cubierto por fichas<br />

de dominó. Probar <strong>que</strong> en a <strong>lo</strong> sumo n(n+1)(2n+1)<br />

6<br />

rotaciones podemos conseguir <strong>que</strong><br />

todos <strong>lo</strong>s dominós estén horizontales.<br />

2. Se tiene un tablero con m fi<strong>la</strong>s y n columnas, con n par, cubierto por fichas de<br />

dominó. Probar <strong>que</strong> mediante “rotaciones“ se puede hacer <strong>que</strong> todas <strong>la</strong>s fichas <strong>que</strong>den<br />

horizontales.<br />

3. Demostrar <strong>que</strong> hay exactamente 2 n(n+1)<br />

2 formas de cubrir al Diamante Azteca de orden<br />

n con fichas de dominó.<br />

4. Probar <strong>que</strong> <strong>para</strong> todo cubrimiento T de rango r(T ) de un Diamante Azteca de orden<br />

n vale <strong>que</strong>:<br />

n(n + 1)(2n + 1)<br />

0 ≤ r(T ) ≤<br />

6<br />

Hal<strong>la</strong>r <strong>lo</strong>s casos donde se da <strong>la</strong> igualdad.<br />

5. Probar <strong>que</strong> desde cualquier cubrimiento de un Diamante Azteca se puede hacer una<br />

rotación <strong>que</strong> baje en 1 el rango. Usar esto y el problema anterior <strong>para</strong> dar otra demostración<br />

del problema 1.<br />

6. Supongamos <strong>que</strong> tenemos n puntos numerados 1, 2, 3, ..., n. Un árbol T es una colección<br />

de n − 1 segmentos entre <strong>lo</strong>s n puntos de forma <strong>que</strong> desde cualquier punto se pueda<br />

llegar a cualquier otro viajando por <strong>lo</strong>s segmentos. En <strong>la</strong> siguiente suma, T recorre<br />

todos <strong>lo</strong>s árboles en estos n puntos y <strong>para</strong> cada uno de el<strong>lo</strong>s d i (T ) es <strong>la</strong> cantidad de<br />

segmentos <strong>que</strong> usan al punto i de vértice.<br />

P (x 1 , ...., x n ) = ∑ T<br />

x d 1(T )−1<br />

1 x d 2(T )−1<br />

2 ... x<br />

dn(T )−1<br />

n<br />

Probar <strong>que</strong> P (x 1 , ..., x n ) = (x 1 + ... + x n ) n−2 . En particu<strong>la</strong>r hay n n−2 árboles en n<br />

puntos numerados. ¿Cuántos hay con d 1 , d 2 , ..., d n segmentos con vértices en 1, 2, .., n<br />

respectivamente?<br />

58


2.15. Pulgas<br />

Para terminar, un hermoso problema de <strong>la</strong> 41 Imo del 2000. El problema es realmente<br />

bueno.<br />

Sea n ≥ 2 un número entero positivo. Inicialmente hay n pulgas en una recta<br />

horizontal y no todas están en el mismo punto. Para un número real positivo λ,<br />

definimos un salto como sigue:<br />

Se eligen 2 pulgas situadas en puntos A y B con A a <strong>la</strong> izquierda de B. Luego<br />

<strong>la</strong> pulga situada en A salta hasta el punto C de <strong>la</strong> recta, a <strong>la</strong> derecha de B, tal<br />

<strong>que</strong> BC<br />

AB = λ.<br />

Determinar todos <strong>lo</strong>s va<strong>lo</strong>res de λ tales <strong>que</strong> <strong>para</strong> cualquier punto M de <strong>la</strong> recta<br />

y cualesquiera posiciones iniciales de <strong>la</strong>s n pulgas, existe una sucesión finita de<br />

saltos <strong>que</strong> permite situar a todas <strong>la</strong>s pulgas a <strong>la</strong> derecha de M.<br />

En primer lugar notemos <strong>que</strong> alcanza con situar una so<strong>la</strong> pulga a <strong>la</strong> derecha de M ya<br />

<strong>que</strong> después hacemos saltar al resto sobre esta pulga y <strong>lo</strong>gramos poner todas a <strong>la</strong> derecha<br />

de M.<br />

La primer pulga es <strong>la</strong> <strong>que</strong> está más a <strong>la</strong> derecha y <strong>la</strong> última es <strong>la</strong> <strong>que</strong> está más hacia <strong>la</strong><br />

izquierda (<strong>lo</strong>s nombres de <strong>la</strong>s pulgas pueden cambiar después de cada salto). Si tenemos 2<br />

posiciones <strong>para</strong> n pulgas, digamos en <strong>lo</strong>s puntos p 1 , ..., p n <strong>la</strong> primera y p ∗ 1 , ..., p∗ n <strong>la</strong> segunda<br />

con p n y p ∗ n <strong>la</strong>s pulgas más a <strong>la</strong> derecha respectivamente, vamos a decir <strong>que</strong> <strong>la</strong> segunda<br />

posición avanzó d con respecto a <strong>la</strong> primera si p ∗ n está a <strong>la</strong> derecha de p n y p n p ∗ n = d.<br />

Definamos el combustible de un conjunto de pulgas como <strong>la</strong> suma de <strong>la</strong>s distancias de<br />

<strong>la</strong> primer pulga a <strong>la</strong>s otras n − 1 pulgas. A continuación vamos a analizar qué pasa con<br />

el combustible después de cada salto. Supongamos entonces <strong>que</strong> antes del i−ésimo salto el<br />

combustible es C i y después de éste <strong>la</strong>s pulgas avanzan d i .<br />

Si <strong>la</strong> primera pulga sigue siendo <strong>la</strong> misma, es decir d i = 0, es c<strong>la</strong>ro <strong>que</strong> el combustible<br />

disminuye:<br />

C i − C i+1 > 0 (1)<br />

En caso contrario, supongamos <strong>que</strong> una pulga en A salta hasta C superando a <strong>la</strong><br />

primer pulga <strong>que</strong> está en B. Entonces d i = BC y BC<br />

BA<br />

≤ λ (con igualdad si y só<strong>lo</strong> si <strong>la</strong><br />

pulga saltó sobre <strong>la</strong> primera). Entonces<br />

C i − C i+1 = BA − BC − (n − 2)BC<br />

59


C i − C i+1 = BC( BA − (n − 1))<br />

BC<br />

C i − C i+1 ≥ d i ( 1 − (n − 1)) (2)<br />

λ<br />

Nos dividimos en 2 casos<br />

1. Si λ < 1<br />

n−1 entonces λ∗ = 1 λ<br />

− (n − 1) > 0. La idea es entonces <strong>que</strong> <strong>para</strong> avanzar d<br />

deben gastar al menos dλ ∗ combustible y como éste es finito no pueden avanzar tanto<br />

como quieran. Para ser más precisos, (1) y (2) se resumen en<br />

Pues en (1) podemos tomar d i = 0, luego<br />

O <strong>lo</strong> <strong>que</strong> es <strong>lo</strong> mismo:<br />

C i − C i+1 ≥ d i λ ∗<br />

C 1 = C 1 − C 2 + C 2 − C 3 + ... + C i − C i+1 + C i+1<br />

C 1 ≥ d 1 λ ∗ + d 2 λ ∗ + ... + d i λ ∗ + C i+1<br />

C 1 ≥ (d 1 + d 2 + ... + d i )λ ∗<br />

d 1 + .... + d i ≤ C 1<br />

λ ∗<br />

Pero después de <strong>lo</strong>s primeros “i“ saltos <strong>la</strong>s pulgas avanzaron, en total, d 1 +d 2 +...+d i .<br />

Es decir <strong>que</strong> <strong>la</strong>s pulgas nunca podrán avanzar más <strong>que</strong> C 1<br />

λ ∗ .<br />

2. Si λ ≥ 1<br />

n−1<br />

entonces hacemos <strong>que</strong> en el i−ésimo salto <strong>la</strong> última pulga salte sobre <strong>la</strong><br />

primera (esto garantiza <strong>que</strong> se da <strong>la</strong> igualdad en (2)). Luego C i ≤ C i+1 ya <strong>que</strong><br />

C i − C i+1 = d i ( 1 − (n − 1)) ≤ 0<br />

λ<br />

Por otro <strong>la</strong>do d i ≥<br />

C i<br />

n−1λ, pues antes del i−ésimo salto <strong>la</strong> primer pulga y <strong>la</strong> última<br />

estaban a distancia ≥ C i<br />

n−1 (por <strong>la</strong> definición de C i). Luego d i ≥ C i<br />

n−1 λ ≥ C 1<br />

n−1 λ.<br />

Como d i ≥ C 1<br />

n−1λ <strong>para</strong> todo “i“ entonces <strong>la</strong>s pulgas avanzan algo fijo en cada salto<br />

entonces tarde o temprano superarán cualquier punto M de <strong>la</strong> recta.<br />

Hemos probado entonces <strong>que</strong> <strong>la</strong>s pulgas pueden ubicarse todas a <strong>la</strong> derecha de cualquier<br />

punto M de <strong>la</strong> recta si y só<strong>lo</strong> si λ ≥ 1<br />

n−1 . ‡ 60


3. Apéndices<br />

61


3.1. Sumas infinitas<br />

“Los ignorantes suponen <strong>que</strong> infinitos sorteos requieren un tiempo infinito; en realidad<br />

basta <strong>que</strong> el tiempo sea infinitamente subdivisible, como <strong>lo</strong> enseña <strong>la</strong> famosa parábo<strong>la</strong> del<br />

Certamen con <strong>la</strong> Tortuga“ J.L.Borges: La Lotería de Babi<strong>lo</strong>nia<br />

Pensemos en Aquiles y <strong>la</strong> Tortuga. Luego de <strong>que</strong> Aquiles trate sin éxito de atrapar<strong>la</strong>,<br />

ésta se apiada del guerrero y decide esperar<strong>lo</strong>. Desde el momento en <strong>que</strong> <strong>la</strong> Tortuga deja<br />

de escapar, Aquiles debe primero alcanzar el punto medio del camino <strong>que</strong> <strong>lo</strong>s se<strong>para</strong>. Una<br />

vez hecho esto, debe alcanzar el punto medio de <strong>la</strong> segunda mitad del camino y luego el<br />

punto medio de <strong>la</strong> segunda mitad de <strong>la</strong> segunda mitad del camino y así indefinidamente. Si<br />

el camino medía 2, y como sabemos <strong>que</strong> Aquiles alcanza <strong>la</strong> tortuga, podemos decir <strong>que</strong><br />

1 + 1 2 + 1 4 + .... + 1 + .... = 2 (1)<br />

2n Pero..... ¿Qué es <strong>lo</strong> <strong>que</strong> justifica <strong>la</strong> igualdad anterior? Después de todo, só<strong>lo</strong> sabemos<br />

sumar finitos números. Lo <strong>que</strong> podemos hacer es ir calcu<strong>la</strong>ndo <strong>la</strong>s “sumas parciales“ y tratar<br />

de ver a qué se parecen.<br />

S 0 = 1 = 1<br />

S 1 = 1 + 1 2 = 1, 5<br />

S 2 = 1 + 1 2 + 1 4<br />

= 1, 75<br />

S 3 = 1 + 1 2 + 1 4 + 1 8<br />

= 1, 875<br />

S 4 = 1 + 1 2 + 1 4 + 1 8 + 1 16<br />

= 1, 9375<br />

Como se observa, <strong>la</strong>s sumas parciales se van acercando cada vez más a 2. En efecto, como<br />

<strong>para</strong> pasar de S n a S n+1 sumamos <strong>la</strong> mitad de <strong>lo</strong> <strong>que</strong> nos falta <strong>para</strong> llegar a 2, entonces<br />

|2 − S n+1 | = 1 2 |2 − S n|<br />

Esto justifica <strong>que</strong> <strong>la</strong>s sumas parciales están cada vez más cerca de 2, tan cerca como <strong>que</strong>ramos,<br />

y de allí <strong>que</strong> (1) tiene sentido.<br />

Ahora tratemos de generalizar <strong>lo</strong> anterior y <strong>para</strong> x tal <strong>que</strong> |x| < 1 vamos a calcu<strong>la</strong>r<br />

1 + x + x 2 + .... + x n + ....<br />

Lo primero <strong>que</strong> haremos es buscar una fórmu<strong>la</strong> <strong>para</strong> <strong>la</strong>s sumas parciales, esto es 1+x+...+x n<br />

x(x n + .... + x + 1) = x n+1 + x n + ... + x<br />

62


x(x n + .... + x + 1) = x n+1 + x n + ... + x + 1 − 1<br />

x(x n + .... + x + 1) = x n+1 − 1 + x n + ... + x + 1<br />

O <strong>lo</strong> <strong>que</strong> es <strong>lo</strong> mismo<br />

(x − 1)(x n + .... + x + 1) = x n+1 − 1<br />

x n + .... + x + 1 = xn+1 − 1<br />

x − 1<br />

x n + .... + x + 1 = 1 − xn+1<br />

1 − x<br />

Ahora analicemos qué pasa con (2) cuando n se va haciendo más y más grande. Como<br />

|x| < 1, entonces x n+1 se hace cada vez más chico (tanto como <strong>que</strong>ramos). Luego <strong>para</strong><br />

va<strong>lo</strong>res grandes de n, el numerador en (2) es muy parecido a 1 y como el denominador es<br />

siempre el mismo, podemos decir <strong>que</strong> <strong>la</strong>s sumas parciales son cada vez más parecidas a 1<br />

1−x .<br />

Pongamos entonces<br />

1 + x + x 2 + .... + x n + .... = 1<br />

1 − x<br />

Recordemos <strong>que</strong> <strong>la</strong> fórmu<strong>la</strong> anterior so<strong>la</strong>mente vale <strong>para</strong> |x| < 1. Só<strong>lo</strong> <strong>para</strong> asegurarnos<br />

<strong>que</strong> le dimos el va<strong>lo</strong>r correcto, calculemos <strong>que</strong> tan cerca está <strong>la</strong> n−ésima suma parcial<br />

S n = 1 + x + ... + x n de S = 1<br />

1−x :<br />

S − S n = 1<br />

1 − x − (1 + x + ... + xn )<br />

S − S n = 1<br />

1 − x − 1 − xn+1<br />

1 − x<br />

S − S n = xn+1<br />

1 − x<br />

Como |x| < 1, entonces S−S n se va haciendo cada vez más chico. Resumiendo, repitamos<br />

<strong>la</strong>s 2 fórmu<strong>la</strong>s <strong>que</strong> hemos visto:<br />

(2)<br />

y si |x| < 1 entonces<br />

1 + x + .... + x n = 1 − xn+1<br />

1 − x<br />

Por ejemp<strong>lo</strong>, si x = 1 2 entonces<br />

1 + x + .... + x n + .... = 1<br />

1 − x<br />

1 + 1 2 + 1 4 + .... + 1<br />

2 n + ... = 1<br />

1 − 1 2<br />

63


1 + 1 2 + 1 4 + .... + 1<br />

2 n + ... = 2<br />

Si x = 1 3 entonces: 1 + 1 3 + 1 9 + .... + 1<br />

3 n + ... = 1<br />

1 − 1 3<br />

1 + 1 3 + 1 9 + .... + 1<br />

3 n + ... = 3 2<br />

Por último si w es tal <strong>que</strong> w 2 + w = 1 entonces<br />

1 + w + w 2 + ... + w n + ... = 1<br />

1 − w<br />

1 + w + w 2 + ... + w n + ... = 1 w 2<br />

64


3.2. Sumas de potencias<br />

Es conocida <strong>la</strong> historia de cómo Gauss sumó <strong>lo</strong>s números del 1 al 100 con una simple<br />

cuenta. Lo primero <strong>que</strong> hizo fue escribir<strong>lo</strong>s en una fi<strong>la</strong> de menor a mayor. Después <strong>lo</strong>s volvió<br />

a escribir en fi<strong>la</strong>, debajo de <strong>la</strong> primera, pero esta vez de mayor a menor.<br />

1 2 3 ..... 99 100<br />

100 99 .. ..... 2 1<br />

En cada columna <strong>lo</strong>s 2 números suman 101, hay 100 columnas de donde <strong>lo</strong>s números<br />

deben sumar 100×101. Como cada número está escrito dos veces, <strong>la</strong> suma deseada es 100×101<br />

2<br />

.<br />

De forma más general, con <strong>la</strong> misma idea podemos ver <strong>que</strong>:<br />

n × (n + 1)<br />

1 + 2 + .... + n = (1)<br />

2<br />

Otra manera de probar (1) es por inducción. La fórmu<strong>la</strong> es cierta <strong>para</strong> n = 1, ahora<br />

supongámos<strong>la</strong> cierta <strong>para</strong> n = k y veámos<strong>lo</strong> <strong>para</strong> n = k + 1.<br />

1 + 2 + ... + (k + 1) = (1 + 2 + ... + k) + k + 1<br />

k(k + 1)<br />

1 + 2 + ... + (k + 1) = + k + 1<br />

2<br />

k(k + 1) + 2(k + 1)<br />

1 + 2 + ... + (k + 1) =<br />

2<br />

(k + 1)(k + 2)<br />

1 + 2 + ... + (k + 1) =<br />

2<br />

Como <strong>la</strong> fórmu<strong>la</strong> vale <strong>para</strong> n = 1, por <strong>lo</strong> <strong>que</strong> acabamos de probar también debe valer<br />

<strong>para</strong> n = 2 y entonces también <strong>lo</strong> hace <strong>para</strong> n = 3 y así vemos <strong>que</strong> vale <strong>para</strong> todo n.<br />

De <strong>la</strong> misma forma <strong>que</strong> acabamos de probar (1) podemos demostrar:<br />

1 2 + 2 2 + ... + k 2 k(k + 1)(2k + 1)<br />

= (2)<br />

6<br />

En efecto, <strong>la</strong> fórmu<strong>la</strong> vale <strong>para</strong> n = 1, supongámos<strong>la</strong> cierta <strong>para</strong> n = k y veámos<strong>lo</strong> <strong>para</strong><br />

n = k + 1<br />

1 2 + 2 2 + ... + (k + 1) 2 = (1 2 + 2 2 + ... + k 2 ) + (k + 1) 2<br />

1 2 + 2 2 + ... + (k + 1) 2 k(k + 1)(2k + 1)<br />

= + (k + 1) 2<br />

6<br />

1 2 + 2 2 + ... + (k + 1) 2 (k + 1)(k(2k + 1) + 6(k + 1))<br />

=<br />

6<br />

1 2 + 2 2 + ... + (k + 1) 2 (k + 1)(k + 2)(2k + 3)<br />

=<br />

6<br />

65


Usando <strong>la</strong>s identidades anteriores podemos probar otras más complicadas:<br />

k=1<br />

n∑<br />

(2k − 1)(n − k + 1) =<br />

k=1<br />

n∑<br />

(2k − 1)(n − k + 1) =<br />

k=1<br />

n∑<br />

2kn − 2k 2 + 2k − n + k − 1<br />

k=1<br />

n∑<br />

(2n + 3)k − 2k 2 − (n + 1)<br />

k=1<br />

n∑<br />

(2k − 1)(n − k + 1) = (2n + 3)<br />

k=1<br />

n∑<br />

n(n + 1)<br />

(2k − 1)(n − k + 1) = (2n + 3)<br />

2<br />

n∑<br />

(2k − 1)(n − k + 1) =<br />

k=1<br />

n∑<br />

(2k − 1)(n − k + 1) =<br />

k=1<br />

n∑<br />

k − 2<br />

k=1<br />

−<br />

n∑<br />

k 2 − n(n + 1)<br />

k=1<br />

2n(n + 1)(2n + 1)<br />

6<br />

− n(n + 1)<br />

3(2n + 3)n(n + 1) − 2n(n + 1)(2n + 1) − 6n(n + 1)<br />

6<br />

n∑<br />

(2k − 1)(n − k + 1) =<br />

k=1<br />

n(n + 1)(3(2n + 3) − 2(2n + 1) − 6)<br />

6<br />

n(n + 1)(2n + 1)<br />

6<br />

66


3.3. Sucesiones recurrentes<br />

La sucesión de Fibonacci comienza con 0 y 1 y cada término se obtiene como <strong>la</strong> suma<br />

de <strong>lo</strong>s 2 anteriores.<br />

0, 1, 1, 2, 3, 5, 8, 13, 21, 34, 55, 89, 144, 233, 377, 610<br />

Si l<strong>la</strong>mamos F n al n−ésimo término comenzando por F 0 = 0, F 1 = 1 entonces {F n }<br />

satisface:<br />

F n+2 = F n+1 + F n<br />

Nuestro objetivo es dar una fórmu<strong>la</strong> <strong>para</strong> F n . Para el<strong>lo</strong> primero hallemos otras sucesiones<br />

más simples <strong>que</strong> satisfagan <strong>la</strong> misma recurrencia. Por ejemp<strong>lo</strong>, bus<strong>que</strong>mos sucesiones de <strong>la</strong><br />

forma:<br />

1, x, x 2 , x 3 , ......, x n , x n+1 , x n+2 , ....<br />

La condición de <strong>que</strong> cada término sea <strong>la</strong> suma de <strong>lo</strong>s 2 anteriores se traduce en <strong>que</strong> <strong>para</strong><br />

todo n<br />

x n+2 = x n+1 + x n<br />

Luego x 2 = x + 1. Resolviendo <strong>la</strong> cuadrática llegamos a 2 posibles va<strong>lo</strong>res, x 1 = 1+√ 5<br />

2<br />

y<br />

x 2 = 1−√ 5<br />

2<br />

. Es decir <strong>que</strong> <strong>para</strong> todo n<br />

( 1 + √ 5<br />

2<br />

( 1 − √ 5<br />

2<br />

) n+2 = ( 1 + √ 5<br />

2<br />

) n+2 = ( 1 − √ 5<br />

2<br />

) n+1 + ( 1 + √ 5<br />

) n<br />

2<br />

) n+1 + ( 1 − √ 5<br />

) n<br />

2<br />

Es fácil ver ahora <strong>que</strong> si ponemos a n = αx n 1 +βxn 2 (recordar <strong>que</strong> x 1 = 1+√ 5<br />

2<br />

y x 2 = 1−√ 5<br />

entonces<br />

2<br />

)<br />

a n+2 = a n+1 + a n<br />

¿Será posible encontrar α y β de forma <strong>que</strong> a n = F n <strong>para</strong> todo n? ¡Sí! Por empezar si<br />

a 0 = F 0 = 0 y a 1 = F 1 = 1 luego<br />

α 1 + √ 5<br />

2<br />

α + β = a 0 = 0<br />

+ β 1 − √ 5<br />

2<br />

= a 1 = 1<br />

Resolviendo el sistema de ecuaciones hal<strong>la</strong>mos <strong>que</strong> α = √ 1<br />

5<br />

y β = − √ 1<br />

5<br />

. Es decir <strong>que</strong><br />

a n = √ 1 ( 1 + √ 5<br />

) n − √ 1 ( 1 − √ 5<br />

5 2 5 2<br />

67<br />

) n


Entonces a n+2 = a n+1 + a n , a 0 = 0 y a 1 = 1. Pero como {a n } y {F n } comienzan igual<br />

y cumplen <strong>la</strong> misma recurrencia entonces deben ser iguales. Es decir <strong>que</strong><br />

F n = √ 1 ( 1 + √ 5<br />

) n − √ 1 ( 1 − √ 5<br />

5 2 5 2<br />

Hagamos otro ejemp<strong>lo</strong> <strong>para</strong> ver cómo funciona esto. Definamos <strong>la</strong> sucesión {G n } por<br />

G 0 = 6 y G 1 = 13 y<br />

) n<br />

G n+2 = 5G n+1 − 6G n<br />

Tratemos de hal<strong>la</strong>r una fórmu<strong>la</strong> <strong>para</strong> G n . Los primeros términos de <strong>la</strong> sucesión son<br />

6, 13, 29, 67, 161, 403, 1049, 2827, 7841, 22243, ....<br />

Las sucesiones de <strong>la</strong> forma {x n } <strong>que</strong> satisfacen <strong>la</strong> misma recurrencia son a<strong>que</strong>l<strong>la</strong>s tales<br />

<strong>que</strong> <strong>para</strong> todo n<br />

O <strong>lo</strong> <strong>que</strong> es <strong>lo</strong> mismo<br />

x n+2 = 5x n+1 − 6x n<br />

x 2 = 5x − 6<br />

Las soluciones de <strong>la</strong> ecuación anterior son x = 2 y x = 3 entonces <strong>que</strong>remos hal<strong>la</strong>r α y<br />

β tales <strong>que</strong><br />

Pero G 0 = 6 y G 1 = 13, esto implica <strong>que</strong><br />

G n = α · 2 n + β · 3 n<br />

α + β = 6<br />

2α + 3β = 13<br />

Resolviendo el sistema anterior llegamos a <strong>que</strong> β = 1 y α = 5. Es decir <strong>que</strong><br />

G n = 5 · 2 n + 3 n<br />

68


3.4. Permutaciones<br />

Una permutación π = (π(1), π(2), ..., π(n)) de 1, 2, ..., n es un reordenamiento de <strong>lo</strong>s<br />

números 1, 2, ..., n. Por ejemp<strong>lo</strong>, si n = 7 <strong>la</strong>s siguientes son permutaciones de 1234567:<br />

6437215, 1234567, 7531246, 4361275, 7654321<br />

Para calcu<strong>la</strong>r cuántas permutaciones de 1, ..., n hay, alcanza notar <strong>que</strong> tenemos n opciones<br />

<strong>para</strong> el primer lugar, luego <strong>que</strong>dan n − 1 opciones <strong>para</strong> el segundo, luego n − 2 <strong>para</strong> el<br />

tercero,...,2 opciones <strong>para</strong> el anteúltimo y 1 opción <strong>para</strong> el último lugar. En definitiva, hay<br />

n(n−1)(n−2).... 3 2 1 = n! permutaciones de 1, 2, ..., n. A continuación listamos <strong>la</strong>s 5! = 120<br />

permutaciones de 1, 2, 3, 4, 5.<br />

12345 14235 21345 24135 31245 34125 41235 43125 51234 53124<br />

12354 14253 21354 24153 31254 34152 41253 43152 51243 53142<br />

12435 14325 21435 24315 31425 34215 41325 43215 51325 53214<br />

12453 14352 21453 24351 31452 34251 41352 43251 51352 53241<br />

12534 14523 21534 24513 31524 34512 41523 43512 51423 53412<br />

12543 14532 21543 24531 31542 34521 41532 43521 51432 53421<br />

13245 15234 23145 25134 32145 35124 42135 45123 52134 54123<br />

13254 15243 23154 25143 32154 35142 42153 45132 52143 54132<br />

13425 15324 23415 25314 32415 35214 42315 45213 52314 54213<br />

13452 15342 23451 25341 32451 35241 42351 45231 52341 54231<br />

13524 15423 23514 25413 32514 35412 42513 45312 52413 54312<br />

13542 15432 23541 25431 32541 35421 42531 45321 52431 54321<br />

Cuadro 1: S 5 = 120 permutaciones de 1,2,3,4,5<br />

Si tenemos una permutación π = (π(1), π(2), ..., π(n)), una “inversión“ es un par i, j tal<br />

<strong>que</strong> i < j y π(i) > π(j). La cantidad de inversiones de π se denota por α(π) y el signo de <strong>la</strong><br />

permutación es sg (π) = (−1) α(π) . Por ejemp<strong>lo</strong><br />

π = (35214) → α(π) = 6 → sg (π) = 1<br />

π = (15342) → α(π) = 5 → sg (π) = −1<br />

π = (14325) → α(π) = 3 → sg (π) = −1<br />

El conjunto de permutaciones de 1, 2, ..., n se l<strong>la</strong>ma S n . En realidad, S n no es so<strong>la</strong>mente un<br />

conjunto sino un “grupo“ pues se puede operar con <strong>la</strong>s permutaciones de forma natural. Para<br />

el<strong>lo</strong> tomemos n letras ordenadas y consideremos cada permutación π = (π(1), π(2), ..., π(n) )<br />

como una orden <strong>que</strong> reordena <strong>la</strong>s n letras de forma <strong>que</strong> <strong>la</strong> primer letra <strong>que</strong>de en el lugar<br />

69


π(1), <strong>la</strong> segunda letra en el lugar π(2), <strong>la</strong> tercera en π(3),...., <strong>la</strong> última en π(n). Por ejemp<strong>lo</strong><br />

si n = 5.<br />

(13425)(ABCDE) = ADBCE<br />

(21345)(ABCDE) = BACED<br />

(51234)(ABCDE) = BCDEA<br />

Cuando aplicamos más de una orden, éstas se leen de derecha a izquierda, por ejemp<strong>lo</strong>:<br />

(12534)(43521)(54321)(ABCDE) = (12534)(43521)(EDCBA)<br />

(12534)(43521)(54321)(ABCDE) = (12534)(ABDEC)<br />

(12534)(43521)(54321)(ABCDE) = (ABECD)<br />

Es fácil deducir “reg<strong>la</strong>s <strong>para</strong> reemp<strong>la</strong>zar“ 2 permutaciones por otra. Por ejemp<strong>lo</strong>:<br />

(12453)(21345)(ABCDE) = (12453)(BACED) = BADCE = (21435)(ABCDE)<br />

(21345)(12453)(ABCDE) = (21345)(ABECD) = BAECD = (21453)(ABCDE)<br />

Luego (12453)(21345) = (21435) y (21345)(12453) = (21453). La ventaja de pensar <strong>la</strong>s<br />

permutaciones como órdenes es <strong>que</strong> podemos operar con el<strong>la</strong>s; si π 1 , π 2 son dos permutaciónes<br />

entonces π 2 π 1 es <strong>la</strong> permutación <strong>que</strong> ordena <strong>la</strong>s letras como <strong>lo</strong> hace primero aplicar π 1 y<br />

después π 2 .<br />

70


3.5. Mene<strong>la</strong>o<br />

Supongamos <strong>que</strong> tenemos un triángu<strong>lo</strong> abc y puntos x, y, z en ab, bc, ca respectivamente.<br />

¿Cómo <strong>saber</strong> si xyz están alineados?<br />

Teorema de Mene<strong>la</strong>o: Sea abc un triángu<strong>lo</strong> y L una recta <strong>que</strong> corta a <strong>lo</strong>s <strong>la</strong>dos ab, bc, ca<br />

en x, y, z respectivamente. Luego<br />

ax by cz<br />

bx cy az = 1<br />

Demostración: Sean h a , h b , h c <strong>lo</strong>s pies de <strong>la</strong>s perpendicu<strong>la</strong>res desde a, b, c a <strong>la</strong> recta L.<br />

Luego <strong>lo</strong>s triángu<strong>lo</strong>s ah a x y bh b x son semejantes de donde<br />

Aná<strong>lo</strong>gamente<br />

ax<br />

bx = ah a<br />

bh b<br />

(1)<br />

by<br />

cy<br />

cz<br />

az<br />

= bh b<br />

ch c<br />

(2)<br />

= ch c<br />

ah a<br />

(3)<br />

71


Multiplicando (1), (2) y (3):<br />

Como <strong>que</strong>ríamos. ‡<br />

ax by cz<br />

bx cy az = ah a bh b ch c<br />

bh b ch c ah a<br />

ax by cz<br />

bx cy az = 1<br />

Ahora veamos el “Dual“ de Mene<strong>la</strong>o, el Teorema de Ceva.<br />

Teorema de Ceva: Sea abc un triángu<strong>lo</strong> y x, y, z puntos en <strong>lo</strong>s <strong>la</strong>dos ab, bc, ca respectivamente<br />

tales <strong>que</strong> cx, ay, bz se cortan en un punto p. Entonces<br />

ax by cz<br />

bx cy az = 1<br />

Demostración: Recordemos <strong>que</strong> si tenemos 2 triángu<strong>lo</strong>s con <strong>la</strong> misma altura, <strong>la</strong> razón<br />

de sus áreas es <strong>la</strong> razón de sus bases. Luego<br />

by<br />

yc = (aby)<br />

(ayc) = (bpy) (aby) − (bpy)<br />

=<br />

(cpy) (ayc) − (cpy) = (abp)<br />

(cap)<br />

Aná<strong>lo</strong>gamente<br />

De donde<br />

by<br />

yc = (bap)<br />

(cap)<br />

cz<br />

za = (cpb)<br />

(apb)<br />

ax<br />

xb = (apc)<br />

(bpc)<br />

72


Como <strong>que</strong>ríamos. ‡<br />

ax by cz<br />

bx cy az = (apc) (bap) (cpb)<br />

(bpc) (cap) (apb)<br />

ax by cz<br />

bx cy az = 1<br />

Es fácil ver <strong>que</strong> valen <strong>lo</strong>s recíprocos de Mene<strong>la</strong>o y Ceva aun<strong>que</strong> con un pe<strong>que</strong>ño detalle,<br />

como no usamos segmentos dirigidos vamos a decir<strong>lo</strong> de <strong>la</strong> siguiente manera.<br />

“Sea abc un triángu<strong>lo</strong> y x, y, z puntos en <strong>lo</strong>s <strong>la</strong>dos ab, bc, ca (o en sus pro<strong>lo</strong>ngaciones)<br />

respectivamente. Supongamos <strong>que</strong><br />

ax by cz<br />

bx cy az = 1<br />

Entonces xyz están alineados o ay, bz, cx se cortan en un punto 16 .“<br />

Hagamos una observación más. Supongamos <strong>que</strong> tenemos un triángu<strong>lo</strong> abc y puntos<br />

x, y, z puntos en <strong>lo</strong>s <strong>la</strong>dos ab, bc, ca respectivamente tales <strong>que</strong> cx, ay, bz se cortan en un<br />

punto p. Mar<strong>que</strong>mos ahora y ′ <strong>la</strong> intersección de <strong>la</strong> recta xz con bc. Luego por el Teorema<br />

de Ceva y el Teorema de Mene<strong>la</strong>o:<br />

ax by cz<br />

bx cy az = 1<br />

ax by ′ cz<br />

bx cy ′ az = 1<br />

16 Si hubiéramos usado segmentos dirigidos entonces se cumple <strong>que</strong> están alineados si el producto<br />

es −1 y son concurrentes si el producto es 1<br />

73


De donde<br />

by<br />

cy = by′<br />

cy ′<br />

Es decir <strong>que</strong> 17 {bc : yy ′ } = 1. Tales puntos se dicen “conjugados harmónicos“ y es una<br />

propiedad invariante por proyección (pues de hecho <strong>la</strong> razón doble <strong>lo</strong> es). Los conjugados<br />

harmónicos tienen <strong>la</strong> siguiente propiedad:<br />

“Sean a, b, c, d cuatro puntos distintos en una recta conjugados harmónicos, es<br />

decir tales <strong>que</strong> {ac : bd} = 1. Tomemos p fuera de <strong>la</strong> recta y cualquier punto q<br />

en <strong>la</strong> recta bp. Luego sea r <strong>la</strong> intersección de cq con ap y s <strong>la</strong> intersección de aq<br />

con cp, entonces r, s, d están alineados.“<br />

17 Para <strong>la</strong> definción de <strong>la</strong> razón doble ir a “Un poco de Geometría Proyectiva“.<br />

74


4. Problemas<br />

75


4.1. Algunos problemas <strong>para</strong> pensar<br />

1. Los números 1, 2, 3, ..., 30 están escritos en un círcu<strong>lo</strong> en un orden arbitrario. En cada<br />

turno se puede intercambiar 2 números consecutivos. Si al final cada número ocupa <strong>la</strong><br />

posición diametralmente opuesta a <strong>la</strong> inicial. Probar <strong>que</strong> en algún momento 2 números<br />

con suma 31 fueron intercambiados.<br />

2. Se dispone de un tablero de una so<strong>la</strong> fi<strong>la</strong> pero infinitas casil<strong>la</strong>s y de un número finito de<br />

porotos <strong>que</strong> se ubican ocupando casil<strong>la</strong>s del tablero. Se realiza una sucesión de movidas<br />

como sigue: en cada paso, se elige una casil<strong>la</strong> <strong>que</strong> tenga más de un poroto; se toman dos<br />

de esos porotos y se co<strong>lo</strong>can uno en <strong>la</strong> casil<strong>la</strong> <strong>que</strong> está inmediatamente a <strong>la</strong> derecha<br />

y el otro en <strong>la</strong> casil<strong>la</strong> <strong>que</strong> está inmediatamente a <strong>la</strong> izquierda de <strong>la</strong> casil<strong>la</strong> elegida.<br />

La sucesión de movidas finaliza si en algún momento hay a <strong>lo</strong> sumo un poroto en<br />

cada casil<strong>la</strong>. Dada una configuración inicial, demostrar <strong>que</strong> toda sucesión de movidas<br />

finalizará después de <strong>la</strong> misma cantidad de pasos y con <strong>la</strong> misma configuración.<br />

3. (San Petersburgo 1997) El número 999..,999 (1997 nueves) está escrito en el pizarrón.<br />

Cada minuto uno de <strong>lo</strong>s números del pizarrón se factoriza como producto de 2 números,<br />

se borra el número y se reemp<strong>la</strong>za por sus 2 factores aumentados o disminuidos en<br />

2 (independientemente). ¿Es posible <strong>que</strong> en algún momento todos <strong>lo</strong>s números del<br />

pizarrón sean iguales a 9?<br />

4. (Moscú 2002) Tenemos un col<strong>la</strong>r con p n per<strong>la</strong>s (p primo y n ≥ 2). En cada operación el<br />

col<strong>la</strong>r se corta en varios pedazos del mismo <strong>la</strong>rgo, se da vuelta el orden de <strong>la</strong>s per<strong>la</strong>s en<br />

cada pedazo y se rearma el col<strong>la</strong>r con <strong>lo</strong>s pedazos en el mismo orden en <strong>que</strong> estaban.<br />

¿Se puede obtener cualquier orden de <strong>la</strong>s per<strong>la</strong>s mediante tales operaciones?<br />

5. (Torneo de Las Ciudades 1980) Se tienen algunos puntos alrededor de un círcu<strong>lo</strong> cada<br />

uno de el<strong>lo</strong>s co<strong>lo</strong>reados de b<strong>la</strong>nco o negro. Se pueden realizar 2 tipos de operaciones:<br />

a) Se cambian de co<strong>lo</strong>r 2 puntos consecutivos y se agrega un punto b<strong>la</strong>nco entre<br />

ambos.<br />

b) Se quita un punto b<strong>la</strong>nco y se cambia el co<strong>lo</strong>r de <strong>lo</strong>s 2 vecinos.<br />

Probar <strong>que</strong> si inicialmente só<strong>lo</strong> hay 2 puntos b<strong>la</strong>ncos no se puede conseguir tener só<strong>lo</strong><br />

2 puntos negros.<br />

6. (Torneo de Las Ciudades 1983) Se tiene un tablero de ajedrez infinito y en cada casil<strong>la</strong><br />

hay un rey salvo por un conjunto de casil<strong>la</strong>s C. Una movida consiste en mover algunos<br />

reyes (posiblemente todos) a una casil<strong>la</strong> vecina (cada casil<strong>la</strong> tiene 8 vecinas) de forma<br />

<strong>que</strong> en cada casil<strong>la</strong> <strong>que</strong>de a <strong>lo</strong> sumo 1 rey.<br />

a) Si C es el conjunto de casil<strong>la</strong>s (m, n) con m y n múltip<strong>lo</strong>s de 100. ¿Es posible<br />

<strong>que</strong> luego de una cantidad finita de movidas cada casil<strong>la</strong> tenga un rey?<br />

76


) Si C es el conjunto de casil<strong>la</strong>s ocupadas o atacadas por un conjunto arbitrario<br />

de damas en el tablero. ¿Es posible <strong>que</strong> luego de una cantidad finita de movidas<br />

cada casil<strong>la</strong> tenga un rey?<br />

7. (Torneo de Las Ciudades 1987) Se tiene un triángu<strong>lo</strong> equilátero de <strong>la</strong>do n dividido<br />

en n 2 triángu<strong>lo</strong>s equiláteros de <strong>la</strong>do 1. Inicialmente hay un triángu<strong>lo</strong> de b<strong>la</strong>nco y el<br />

resto de negro. En cada turno se puede cambiar el co<strong>lo</strong>r de todos <strong>lo</strong>s triángu<strong>lo</strong>s entre<br />

2 <strong>para</strong>le<strong>la</strong>s consecutivas a un <strong>la</strong>do del triángu<strong>lo</strong>. ¿Puede conseguirse <strong>que</strong> todos <strong>lo</strong>s<br />

triángu<strong>lo</strong>s sean negros?<br />

8. (Torneo de Las Ciudades 1993) Hay 4 ranas. En cada turno una rana salta. Si <strong>la</strong> rana<br />

<strong>que</strong> salta estaba en A y <strong>la</strong>s otras en B,C y D entonces <strong>la</strong> rana salta hasta E tal <strong>que</strong><br />

el punto medio de AE sea el baricentro de BCD. Si inicialmente <strong>la</strong>s ranas estaban en<br />

<strong>lo</strong>s vértices de un cuadrado, ¿es posible <strong>que</strong> luego de algunos turnos una rana salte<br />

encima de otra?<br />

9. (Torneo de Las Ciudades 2005) Inicialmente no hay ningún número en el pizarrón,<br />

en cada etapa se puede o bien escribir dos 1’s o bien borrar dos números idénticos,<br />

digamos n y al mismo tiempo escribir <strong>lo</strong>s dos números n + 1 y n − 1. Determinar<br />

el número mínimo de etapas necesarias <strong>para</strong> <strong>que</strong> al menos uno de <strong>lo</strong>s números del<br />

pizarrón sea el 2005.<br />

10. (Leningrado 1988) Se tiene una pi<strong>la</strong> con 1001 piedras. En cada turno se elige una pi<strong>la</strong>,<br />

se le quita una piedra y el resto se divide en 2 pi<strong>la</strong>s nuevas. ¿Es posible <strong>lo</strong>grar <strong>que</strong><br />

todas <strong>la</strong>s pi<strong>la</strong>s tengan 3 piedras?<br />

11. (Leningrado 1988) Cualquier secuencia de 0 ′ s y 1 ′ s es una pa<strong>la</strong>bra. Un triplicado es<br />

tomar una pa<strong>la</strong>bra y repetir<strong>la</strong> 3 veces (una a continuación de <strong>la</strong> otra). Por ejemp<strong>lo</strong><br />

010101, 111, 100110011001. Dada una pa<strong>la</strong>bra se <strong>la</strong> puede transformar en otra<br />

introduciéndole o borrándole un triplicado en cualquier lugar. ¿Es posible cambiar <strong>la</strong><br />

pa<strong>la</strong>bra 01 en 10 por una secuencia de transformaciones como <strong>la</strong>s recién descripta?<br />

12. (Leningrado 1991) Se tienen varios números enteros escritos en un círcu<strong>lo</strong>. En cada<br />

turno se puede quitar un número par y sumar <strong>lo</strong>s 2 números <strong>que</strong> eran vecinos a éste<br />

(.., x, 2y, z, ... → ..., x + y, ...). La operación se repite hasta <strong>que</strong> <strong>que</strong>den 1 o 2 números<br />

o <strong>que</strong> todos <strong>lo</strong>s números sean impares. Probar <strong>que</strong> <strong>la</strong> cantidad de números <strong>que</strong> <strong>que</strong>den<br />

es independiente de <strong>lo</strong> <strong>que</strong> hagamos.<br />

13. (Bamo 2006) Se tienen k manijas alineadas, cada una de <strong>lo</strong>s cuales está hacia arriba,<br />

abajo, derecha o izquierda. Cuando cualesquiera 3 manijas consecutivas apunten en<br />

direcciones distintas se pueden girar <strong>la</strong>s 3 de forma <strong>que</strong> apunten a <strong>la</strong> cuarta dirección.<br />

Probar <strong>que</strong> esta operación no se puede repetir indefinidamente.<br />

14. (Estonia 2002) Sea n un entero positivo dado. Pongamos una ficha en cada número<br />

negativo. En cada turno podemos elegir n enteros consecutivos, quitar una ficha de<br />

77


uno el<strong>lo</strong>s y reacomodar <strong>la</strong>s demás fichas de estos n enteros como <strong>que</strong>ramos en <strong>lo</strong>s n<br />

enteros (a <strong>lo</strong> sumo una ficha por número). Decidir si existe n de forma <strong>que</strong> <strong>para</strong> cada<br />

N > 0 es posible poner una ficha a <strong>la</strong> derecha de N.<br />

15. Nos es dado un tablero de n × n con números enteros en sus casil<strong>la</strong>s. En cada paso<br />

podemos realizar una de <strong>la</strong>s siguientes operaciones:<br />

-Sumar a una fi<strong>la</strong> un múltip<strong>lo</strong> de otra<br />

-Sumar a una columna un múltip<strong>lo</strong> de otra<br />

-Intercambiar 2 fi<strong>la</strong>s<br />

-Intercambiar 2 columnas<br />

Probar <strong>que</strong> se puede llevar al tablero a uno con <strong>lo</strong>s números (d 1 , ..., d n ) en su diagonal<br />

y 0 fuera de el<strong>la</strong> de forma <strong>que</strong> d 1 |d 2 , d 2 |d 3 ,... ,d n−1 |d n . Probar además <strong>que</strong> <strong>lo</strong>s números<br />

d 1 , ..., d n son únicos salvo por su signo (esta es <strong>la</strong> forma normal de Smith).<br />

16. (Tercer problema de Hilbert) ¿Es posible cortar un cubo en una cantidad finita de<br />

poliedros y con el<strong>lo</strong>s armar un tetraedro?<br />

78

Hooray! Your file is uploaded and ready to be published.

Saved successfully!

Ooh no, something went wrong!